Vous êtes sur la page 1sur 55

Electric charges and fields

CHAPTER

16

ELECTRIC CHARGES AND FIELDS

LEARNING OBJECTIVES
(i) Electric and magnetic forces determine the properties of atoms, molecules and bulk matter. Coulomb force and gravitational
force follow the same inverse-square law. But gravitational force has only one sign (always attractive), while Coulomb force can
be of both signs (attractive and repulsive), allowing possibility of cancellation of electric forces. This is how gravity, despite
being a much weaker force, can be a dominating and more pervasive force in nature.
(ii) Charge is not only a scalar (or invariant) under rotation; it is also invariant for frames of reference in relative motion. This is
not always true for every scalar. For example, kinetic energy is a scalar under rotation, but is not invariant for frames of reference
in relative motion. Conservation of total charge of an isolated system is a property independent of the scalar nature of charge.
Conservation refers to invariance in time in a given frame of reference. A quantity may be scalar but not conserved (like kinetic
energy in an inelastic collision). On the other hand, one can have conserved vector quantity (e.g., angular momentum of an
isolated system). Quantisation of electric charge is a basic (unexplained) law of nature; interestingly, there is no analogous law on
quantisation of mass.
(iii) Coulombs Law: The mutual electrostatic force between two point charges q1 and q2 is proportional to the product q1q2 and
inversely proportional to the square of the distance r21 separating them.
1
k (q1q 2 )

r21 where r21 is a unit vector in the direction from q to q and k


Mathematically, F21 = force on q2 due to q1 =
2
1
2
4

0
r21

is the constant of proportionality. In SI units, the unit of charge is coulomb. The experimental value of the constant 0 is
0 8.854 10 12 C 2 N 1m 2 . The approximate value of k is k = 9 109 Nm2C2.
(iv) Superposition principle should not be regarded as obvious, or equated with the law of addition of vectors. It says two
things: force on one charge due to another charge is unaffected by the presence of other charges, and there are no additional
three-body, four-body, etc., forces which arise only when there are more than two charges.
(v) The electric field E at a point due to a charge configuration is the force on a small positive test charge q placed at the point
divided by the magnitude of the charge. Electric field due to a point charge q has a magnitude | q |/40r2 it is radially outwards
from q, if q is positive, and radially inwards if q is negative. Like Coulomb force, electric field also satisfies superposition principle.
The electric field due to a discrete charge configuration is not defined at the locations of the discrete charges. For continuous
volume charge distribution, it is defined at any point in the distribution. For a surface charge distribution, electric field is
discontinuous across the surface.
(vi) An electric field line is a curve drawn in such a way that the tangent at each point on the curve gives the direction of electric
field at that point. The relative closeness of field lines indicates the relative strength of electric field at different points; they crowd
near each other in regions of strong electric field and are far apart where the electric field is weak. In regions of constant electric
field, the field lines are uniformly spaced parallel straight lines. Some of the important properties of field lines are: (i) Field lines
are continuous curves without any breaks. (ii) Two field lines cannot cross each other. (iii) Electrostatic field lines start at positive
charges and end at negative charges they cannot form closed loops.
(vii) An electric dipole is a pair of equal and opposite charges q and q separated by some distance 2a. Its dipole moment vector

p has magnitude 2qa and is in the direction of the dipole axis from q to q.
Field of an electric dipole in its equatorial plane (i.e., the plane perpendicular to its axis and passing through its centre) at a

p
p
1

distance r from the centre: E


,
for r >> a
4 0 (a 2 r 2 ) 3/2
4 0 r 3

Dipole electric field on the axis at a distance r from the centre: E

2pr
4 0 (r 2 a 2 ) 2

2p
4 0 r 3

for r >> a

The 1/r3 dependence of dipole electric fields should be noted in contrast to the 1/r2 dependence of electric field due to a point

charge. In a uniform electric field E , a dipole experiences a torque given by p E but experiences no net force.
GyaanSankalp

Electric charges and fields


(viii) The flux of electric field E through a small area element S is given by E.S

The vector area element S is S S n


where S is the magnitude of the area element and n is normal to the area element, which can be considered planar for sufficiently
small S. For an area element of a closed surface, n is taken to be the direction of outward normal, by convention.
(ix) Gausss law: The flux of electric field through any closed surface S is 1/0 times the total charge enclosed by S. The law is

especially useful in determining electric field E , when the source distribution has simple symmetry:

(a) Thin infinitely long straight wire of uniform linear charge density

n
2 0 r

where r is the perpendicular distance of the point from the wire and n is the radial unit vector in the plane normal to the wire
passing through the point.

(b) Infinite thin plane sheet of uniform surface charge density E 2 n


0

(c) Thin spherical shell of uniform surface charge density E

4 0 r 2

(r R) E 0 (r < R)

INTRODUCTION
Many students feel that in electrostatic charge remains at rest & hence formulae derive in electrostatic are applicable when charge
is at rest but fact is formulae are applicable when charge is in motion only the difference is when charge is in motion we will
consider additional effect called magnetic effect. Electrostatics deals with the study of forces, fields and potentials arising from
static charges. Like mass, electric charge is an intrinsic property of protons and electrons.
In nature, atoms are normally found with equal numbers of protons and electrons, i.e. atom is electrically neutral.
The charge on an electron or a proton is the smallest amount of free charge that has been discovered. Charges of larger magnitude
are built up on an object by adding or removing electrons. If in a body there is excess of electrons over its neutral configuration,
conventionally the body is said to be negatively charged and if there is deficiency of electron it is said to be positively charged.
ve charged body Body has gained electrons
+ ve charged boy Body has lost some electrons
+ ve & ve charge named by benjamin Franklin.
INTERESTING EXPERIMENT
Take any two materials from the following list and then rubbed with each other. We can always find that the former one is
positively charged and the later one is negatively charged.
Fur glass paper metal silk plastic amber rubber sulfur
When a charged body is close enough to a neutral body, they attract each other. One of the applications of this effect is to use tiny
paint droplets to paint the automobiles uniformly.
CONDUCTOR AND INSULATORS
Suppose you charge a rubber rod and then touch it to a neutral object. Some charge, repelled by the negative charge on the rod,
will be transferred to the originally-neutral object. What happens to that charge then depends on the material of which the
originally-neutral object consists. In the case of some materials, the charge will stay on the spot where the originally neutral object
is touched by the charged rod. Such materials are referred to as insulators, materials through which charge cannot move, or,
through which the movement of charge is very limited. Examples of good insulators are quartz, glass, and air. In the case of other
materials, the charge, almost instantly spreads out all over the material, in response to the force of repulsion (recalling that force
causes acceleration which leads to the movement) that each elementary particle of the charge exerts on every other elementary
particle of charge. Materials in which the charge is free to move about are referred to as conductors. Examples of good conductors
are metals and saltwater.
When you put some charge on a conductor, it immediately spreads out all over the conductor. The larger the conductor, the more
it spreads out. In the case of a very large object, the charge can spread out so much that any chunk of the object has a negligible
amount of charge and hence, behaves as if were neutral. Near the surface of the earth, the earth itself is large enough to play such
a role. If we bury a good conductor such as a long copper rod or pipe, in the earth, and connect to it another good conductor such
2

GyaanSankalp

Electric charges and fields


as a copper wire, which we might connect to another metal object, such as a cover plate for an electrical socket, above but near the
surface of the earth, we can take advantage of the earths nature as a huge object made largely of conducting material. If we touch
a charged rubber rod to the metal cover plate just mentioned, and then withdraw the rod, the charge that is transferred to the metal
plate spreads out over the earth to the extent that the cover plate is neutral. We use the expression the charge that was
transferred to the cover plate has flowed into the earth. A conductor that is connected to the earth in the manner that the cover
plate just discussed is connected is called ground. The act of touching a charged object to ground is referred to as grounding
the object. If the object itself is a conductor, grounding it (in the absence of other charged objects) causes it to become neutral.
CHARGING BY INDUCTION
When a charged particle is taken near to neutral metallic object then the electrons move to one side and there is excess of electrons
on that side making it negatively charged and deficiency on the other side making that side positively charged. Hence charges
appear on two sides of the body (although total charge of the body is still zero). This phenomenon is called induction and the
charge produced by it is called induced charge.
+
+ +
+ +
+ +

+
+ +
+ +
+ +

+
+
+
+
+

q'=0
Charged
V'=
+ve
Step 1 : body
Charged body is
brought near
uncharged body

Step 2 :

+
+ +
+ +
+ +

q' = ve
q' = ve
V' = 0
V' = 0
Step 3 :
Uncharged body is
Uncharged body is
disconnected from the
connected to the
earth
earth

Step 4 :

q' = ve
V' = ve
Charging body
is removed

A body can be charged by means of (a) friction, (b) conduction, (c) induction, (d) thermoionic ionisation, (e) photoelectric effect
and (f) field emission.
BASIC PROPERTIES OF ELECTRIC CHARGE
(1) Charge is a scalar and can be of two types (i.e. + ve or ve). It adds algebraically.
(2) Charge is conserved. During any process (chemical, nuclear, decay etc.) the net electric charge of an isolated system remains
constant.
In the process one body gains some amount of ve charge while the other gains an equal amount of + ve charge.
Pair - production, Annihilation are processes understand on basis of charge conservation.
(3) Charge is Quantized (exists as discrete "Packets") : Robert Millikan discovered that electric charge always occurs as some integral
multiple of fundamental unit of charge (e).
q = Ne [N is some integer]

1 2
Charge on a body can never be e, e etc. as it is due to transfer of electron.
3
3
(4) Through large number of experiments it is also well established that similar charges repel each other while dissimilar attract.
Here it is worth noting that true test of electrification is repulsion and not attraction as attraction may also take place between a
charged bodies.
(5) Charge is always associated with mass i.e. charge can not exist without mass though mass can exist without charge.
(6) Charge is transferable. Process of charge transfer is called conduction.
(7) Charge is invariant i.e. it is independent on frame of reference.
(8) Accelerated charge radiates energy.
Charge at rest produces Electric & magnetic effect.
Accelerate charge particle Electric & magnetic effect + radiate energy (According to electromagnetic theory)

v=0

produces E

v = const.

produces E and B
with no radiation

v const.

produces E and B
and radiates energy

(9) Charge resides on the outer surface of a conductor


(10) How to express charge.
The SI unit for measuring the magnitude of an electric charge is the coulomb (C).
Current drift of charge per unit time
I = q/t q = It
GyaanSankalp

Electric charges and fields


1 coulomb 1 ampere 1 sec
If a charge of 1 coulomb drift per second through cross-section of conductor current flowing is called 1 ampere.
Charge on electron = 1.6 1019 C, Charge on proton = + 1.6 1019 C
The coulomb is related to CGS units of charge through the basic relation.
1
emu of charge
10
Practical units of charge are amp hr (= 3600 coulomb), Faraday ( = 96500 coulomb)
Example 1 :
How many electrons are there in one coulomb of negative charge.
Sol. Number N of electrons is

1 Coulomb = 3 109 esu of charge (static coulomb or frankline) =

1.00C
q
=
= 6.25 1018
e
1.6 1019 C
Charge on 6.25 1018 electrons = 1 C
Example 2 :
A copper penny (Z = 29) has a mass of 3g. What is the total charge of all the electrons in the penny ?
Sol. The electrons have a total charge given by the number of electrons in the penny, Ne, times the charge of an electron, e. The
number of electrons is 29 times the number of copper atoms N. To find N, we use the fact that one mole of any substance has
Avogadro's number (NA = 6.02 1023) of molecules, and the number of grams in a mole is the molecular mass M, which is 63.5 for
copper. Since each molecule of copper is just one copper atom, we find the number of atoms per gram by dividing NA atoms/mole
by M grams/mole.
1. The total charge is the number of electrons times the electronic charge : Q = Ne ( e)
2. The number of electrons is Z time the number of copper atoms Na :
Ne = ZNa
N=

3. Compute the number of copper atoms in 3g of copper :

Na = (3g)

6.02 1023 atoms / mol


= 2.84 1022 atoms
63.5g / mol

4. Compute the number of electrons Ne ,Ne = ZNa = (29 electrons/atom) (2.84 1022 atoms) = 8.24 1023 electrons.
5. Use this value of Ne to find the total charge : Q = Ne ( e) = (8.24 1023 electrons) ( 1.6 1019 C/electron) = 1.32 105 C
Example 3 :
A glass rod is rubbed with a silk cloth. The glass rod acquires a charge of + 19.2 1019 C.
(i) Find the number of electrons lost by glass rod.
(ii) Find the negative charge acquired by silk.
(iii) Is there transfer of mass from glass to silk ?
Sol. (i) Number of electrons lost by glass rod is

n=

q 19.2 19 19
=
= 12
e
1.6 10 19

(ii) Charge on silk = 19.2 1019 C


(iii) Since an electron has a finite mass (me = 9 1031 kg), there will be transfer of mass from glass rod to silk cloth.
Mass transferred = 12 (9 1031) = 1.08 1029 kg.
The mass transferred is negligibly small. This is expected because the mass of an electron is extremely small.
DETECTING CHARGE
Charge can be detected and measured with the help of gold-leaf electroscope, voltameter, ballistic galvanometer. Gold leaf
electroscope consist of two gold leaves attached to a conducting post that has a conducting disc ball on top. The leaves are
otherwise insulated from the container. Gold leaf electroscope can be used in 2 ways.
+++
Uncharged electroscope when uncharged, the leaves hang together vertically.
+++
(a) If a charged body is brought near to it, charge on the ball of electroscope will be
opposite to that of body & on leaves similar to that of body and leaves will diverge.
++ ++

(b) If a charged body is touched : Ball & leaves both acquire similar charge and leaves
will diverge.
From above method you will not be able to tell nature of charge (it may be +ve or ve)
in both case leaves will diverge.

GyaanSankalp

+ +++
+++

+
+
+
+
+ ++

Electric charges and fields


Charged electroscope :
If a charged body is brought near a charged electroscope, the leaves
will further diverge if the charge on the body is similar to that on the
electroscope and will usually converge if opposite. Thus we will be
able to determine nature of charge on a body.

+ +++
+++

++ ++

+ +++
+++

++ ++

Example 4 :
What will happen if x-rays are incident on a charged electroscope.
Sol. Due to ionisation of air by x-rays the electroscope will get discharged and hence its leaves will collapse.
Example 5 :
What will happen if x-rays are incident on a charged evaluated electroscope.
Sol. X-rays will cause photoelectric effect with gold and so the leaves will further diverge if it is positively charged (or uncharged) and
will converge if it is negatively charged.

TRY IT YOURSELF
Does the attraction between the comb and the piece of papers last for longer period of time ?
Is repulsion a true test of electrification ?
What is the total charge, in coulombs, of all the electrons in three mole of hydrogen atom ?
The existence of a negative charge on a body implies that it has
(A) Lost some of its electrons
(B) Lost some of its protons
(C) Acquired some electrons from outside
(D) Acquired some protons from outside
Q.5 Lighting rods are made of
(A) Porcelain
(B) Bakelite
(C) Plastic
(D) Metal
Q.6 A positively charged body is brought near an uncharged gold leaf electroscope, then
(A) No charge is induced in the leaves
(B) Positive charge is induced in both the leaves
(C) Negative charge is induced in both the leaves
(D) Positive charge is induced in one leaf and negative in the other.
Q.7 Static electricity is produced by
(A) Friction only
(B) Induction only
(C) Friction & induction both (D) Chemical reaction only
Q.8 Five balls respectively from 1 to 5 are suspended from different threads. If pair of balls (1,2), (2,4) and (4,1) represents attraction
while pair (2,3) and (4,5) represents repulsion then on ball 1.
(A) Positive charge
(B) Negative
(C) Neutral
(D) Made of metal
Q.9 On charging two metallic spheres of same mass(A) Mass of positively charged sphere will be more
(B) Mass of positively charged sphere will be less
(C) Mass of negatively charged sphere will be more
(D) Mass of negatively charged will be less
(A) 1, 2
(B) 2, 3
(C) 3, 4
(D) 1, 4
Q.10 The current produced in wire when 107 electron/sec. are flowing in it(A) 1.6 x 1026 A
(B) 1.6 x 1012 A
(C) 1.6 x 1026 A
(D) 1.6 x 1012 A
ANSWERS
(4) (C)
(5) (D)
(6) (B)
(7) (C)
(8) (C)
(9) (B)
(10) (D)
Q.1
Q.2
Q.3
Q.4

COULOMB'S LAW
Force between two point charges (interaction force) is directly proportional to the product of magnitude of charges (q1 and q2)
and is inversely proportional to the square of the distance between them i.e., (1/r2). This force is conservative in nature. This is
also called inverse square law. The direction of force is always along the line joining the point charges.
F
K=

q1q 2
r

1
4 0 r

F= K

q1q 2
r2

where K is a constant

[K = 9 109 C2/N-m2 ] ;

0 = permittivity of free space = 8.85 1012 N-m2/C2, r = relative permittivity (dielectric constant of medium)
Coulombs Law in Vector Form
Suppose the position vectors of two charges q1 and q2 are

r1 and r2 , then, electric force on charge q1 due to charge q2 is,


GyaanSankalp

Electric charges and fields

F12

q1q 2
1

r1 r2
4 0 | r1 r2 |3

Similarly, electric force on q2 due to charge q1 is

F21

q1q 2
1
3 r2 r1
4 0 | r2 r1 |

Here q1 and q2 are to be substituted with sign. Position vector of

charges q1 and q2 are r1 x1i y1j z1k and r2 x 2i y2 j z 2 k


respectively. Where (x1, y1, z1) and (x2, y2, z2) are the co-ordinates of
charges q1 and q2.
Superposition Theorem
The interaction between any two charges is independent of the presence of all other
charges. Electrical force is a vector quantity therefore, the net force on any one
charge is the vector sum of the all the forces exerted on it due to each of the other
charges interacting with it independently i.e.

Net force on charge q,
F = F1 + F2 + F3 + ...........
Example 6 :
In a hydrogen atom, the electron is separated from the proton by an average distance of about 5.3 1011 m. Calculate the
magnitude of the electrostatic force of attraction exerted by the proton on the electron.
Sol. Substitute the given value into Coulomb's law :
F=

k | q1q 2 |
r2

ke 2
r2

(8.99 109 N.m 2 / C2 ) (1.6 10 19 C) 2

(5.3 1011 m)2

= 8.19 109 N

Example 7 :
Compute the ratio of the electric force to the gravitational force exerted by a proton on an electron in a hydrogen atom.
Sol. We use Coulomb's law with q1 = e and q2 = e to find the electric force, and Newton's law of gravity with the mass of the proton,
mp = 1.67 1027 kg, and the mass of the electron, me = 9.11 1031 kg.
1. Express the magnitude of the electric force Fe and the gravitational force Fg in terms of the charges, masses, separation distance
r, and electrical and gravitational constant s:
Fe =

ke 2
2

, Fg =

Gm p m e

r
r2
2. Take the ratio. Note that the separation distance r cancels :
Fe
ke 2
Fg = Gm p m e

3. Substitute numerical values :


Fe
(8.99 109 N.m 2 / C2 ) (1.6 1019 C)2
39
Fg = (6.67 1011 N.m 2 / kg 2 ) (1.67 1027 kg) (9.11 10 31 kg) = 2.27 10

Example 8 :
What is the smallest electric force between two charges placed at a distance of 1.0 m.

q1q 2
1
. 2
.............. (i)
r
4 0
For Fe to be minimum q1 q2 should be minimum.
We know that
(q1)min = (q2)min = e = 1.6 1019 C
Substituting in Eq. (i), we have

Sol. Fe =

(9.0 109 ) (1.6 1019 ) (1.6 10 19 )

(Fe)min =

GyaanSankalp

(1.0)2

= 2.304 1028 N.

Electric charges and fields


Example 9 :
Electric force between two point charges q and Q at rest is F. Now if a charge q is placed next to q what will be the (a) force on
Q due to q (b) total force on Q ?
Sol. (a) As electric force between two body interaction, i.e., force between two particles, is independent of presence or absence of
other particles, the force between Q and q will remain unchanged, i.e., F.
(b) An electric force is proportional to the magnitude of charges, total force on Q will be given by :

F Qq q 0

0 [as q' = q + ( q) = 0]
F Qq
q q
i.e., The resultant force on Q will be zero.
Example 10 :
Four identical point charges each of magnitude q are placed at the corners of a square of side a. Find the net electrostatic force on
any of the charge.
Sol. Let the concerned charge be at C then charge at C will experience the force due to charges at A, B and D. Let these forces

respectively be FA , FB and FD thus forces are given as

FA

FB

FD

1
q2
q2
along
AC
=
4 0 2a 2
4 0 AC 2

j
i

2
2

B
q

A
q

q
1
q
( j)
along BC =
2
2
4 0 BC
4 0 a

C q

Dq

FB

q
1
q2
(i )
along DC =
2
4 0 DC
4 0 a 2

Fnet FA FB FD

Fnet

q2
4 0 a 2

FD

FA

1
1

1 j
1
i
2 2

2 2

2
1
q2
1
q

2
2
1

4 a 2
2
4 0 a 2
2 2

Example 11 :
Five point charges, each of value + q are placed on five vertices of a regular hexagon of side L m. What is the magnitude of the
force on a point charge of value q coulomb placed at the centre of the hexagon ?
Sol. If there had been a sixth charge + q at the remaining vertex of hexagon force due to all the six charges on q at O will be zero (as

the forces due to individual charges will balance each other), i.e., FR 0

Now if f is the force due to sixth charge and F due to remaining five charges,

F f 0 i.e., F f

or

1
1 q 2
qq
F = f = 4
=
4 0 L
L2
0
+ +

1 q2
(B)
4 0 d 2

+ +

1 q2
(A)
4 0 d 2

+ + + +

Example 12 :
Two charged spheres of radius 'R' are kept at a distance 'd' (d > 2R).
One has a charge +q and the other q. The force between them will be-

1 q
(D) None of these
4 0 d 2
Sol. (B). Redistribution of charge will take place due to mutual attraction and hence effective distance will be less than d.
(C)

GyaanSankalp

Electric charges and fields


TRY IT YOURSELF
Q.1 Two identical balls each having a density are suspended from a common point by two insulating strings of equal length. Both
the balls have equal mass and charge. In equilibrium each string makes an angle with vertical. Now, both the balls are immersed
in a liquid. As a result the angle does not change. The density of the liquid is . Find the dielectric constant of the liquid.
Q.2 The ratio of gravity force to the electric force between two electrons(A) 1036
(B) 1042
(C) 1042
(D) 1047
Q.3 A charge Q1 exerts some force on a second charge Q2. If a 3rd charge Q3 is brought near, the force of Q1 exerted on Q2
(A) Will increase
(B) Will decrease
(C) Will remain unchanged
(D) Will increase if Q3 is of the same sign as Q1 and will decrease if Q3 is of opposite sign.
Q.4 Two identical pendulums A and B, are suspended from the same point. The bobs are given positive charges, A having more charge

Q.5
Q.6
Q.7

Q.8

Q.9

than B. They diverge and reach equilibrium with A and B making angles 1 and 2 with the vertical respectively. Then
(A) 1 > 2
(B) 1 < 2
(C) 1= 2
(D) The tension in A is greater than tension in B
Force of attraction between two point charges placed at a distance d is F. What distance apart should they be kept in the same
medium so that the force between them is F/3 ?
A particle of mass m carrying charge +q1 is revolving around a fixed charge q2 in a circular path of radius r. Calculate the period
of revolution.
Two pieces of copper, each weighing 0.01 kg, are placed at a distance of 0.1m from each other. One electron from per 1000 atoms
of one piece is transferred to other piece of copper. What will be the Coulomb force between two pieces after the transfer of
electrons ? The atomic weight of copper is 63.5 g/mole. Avogadro number = 6 1023.
Two point charges of +2C and +6C repel each other with a force of 12 N. If each is given an additional charge 4C, then force
will become(A) 4 N (attractive)
(B) 60 N (attractive)
(C) 4 N (Repulsive)
(D) 12 N (attractive)
Three equal charges (q) are placed at corners of a equilateral triangle. The force on any charge is (A) zero

(B) 3

Kq 2

(C)

Kq 2

(D) 3 3

Kq 2

3a 2
a2
a2
Q.10 Five point charges, each of value q coulomb, are placed on five vertices of a regular hexagon of side L meter. The magnitude of
the force on a point charge of value q coul. placed at the center of the hexagon is -

(A)

kq 2

(B) 5

kq 2

(C) 3

kq 2
L2

(D) zero

ANSWERS
(1) K = / d
(6) T 4r

0 mr
q1q 2

(2) (B)

(3) (C)

(4) (C)

(5) 1.732 d

(7) F = 2.06 1014 N

(8) (C)

(9) (B)

(10) (A)

ELECTRIC FIELD
The physical field where a charged particle, irrespective of the fact whether it is in motion or at rest, experiences force is called an
electric field. The concept of electric field was given by michael Faraday. Characteristics of electric field :
(1) Electric field intensity (shortly we will call electric field).
(2) Electric potential.
(3) Electric lines of forces.

Electric field intensity E :


Electric field intensity at a point is equal to the electrostatic force experienced by a unit positive point charge both in magnitude
and direction

If a test charge q0 is placed at a point in an electric field and experiences a force F due to some charges (called source charges),

F
the electric field intensity at that point due to source charges is given by E
.
q0
The presence of the charge q0 will generally change the original distribution of the other charges, particularly if the charges are
on conductors. However, we may choose q0 to be small enough so that its effect on the original charge distribution is negligible.

E
E lim
q 0 q0
0
8

GyaanSankalp

Electric charges and fields


Electric field due to a point charge
The electric field produced by a point charge q can be obtained in general terms from Coulomb's law. First, note that the magnitude
of the force exerted by the charge q on a test charge q0 is F = kqq0/r2. Then, divide this value by q0 to obtained the magnitude of
the field. Since q0 is eliminated algebraically from the result, the electric field does not depend on the test charge:
Point charge q :

E=

kq

r2
If (x, y, z) are the co-ordinates of the observation point P, then

r xi yj zk

q0
P

Also, r = (x2 + y2 + z2)1/2 and r3 = (x2 + y2 + z2)3/2


Now, E(r) =

q
1

(xi yj zk)
2
2
4 0 (x y z 2 )3/ 2

The three rectangular components of E (r) are as follows :

Ex( r ) =

q
1
x,
2
2
4 0 (x y z 2 )3/ 2

Ey( r ) =

q
1
y and
4 0 (x 2 y 2 z 2 )3/ 2

Ez( r ) =

O
q Source Charge

q
1
z
2
2
4 0 (x y z 2 )3/ 2

Electric field due to Discrete distribution of charge :


Point charges placed at different position, use vector approach (Better term : super position rule)

E E1 E 2 ....... Ei

with E i

i 1

1 qi
ri
4 0 ri3

Corona Discharge : Dielectric strength of medium mean minimum field required for ionisation of a medium. If value of E increases
above dielectric strength of medium, medium gets ionised and charge leak out into the medium from body generally it happen at
the corner where E is high. This leakage process is called corona discharge.
For air dielectric strength = 3 106 v/m
The electric field near a high-voltage power line can be large enough to strip the electrons from air molecules, thus ionizing them
and making the air a conductor. The glow resulting from the recombination of free electrons with the ions is an example of corona
discharge. Break-down in air is witnessed during atmospheric lighting.
Motion of a charged particle in a uniform electric field
If force of gravity does not exist or is balanced by some other force say reaction or neglected then

F qE
a
constant [as F qE ]
m m
Here equations of motion are valid.

(i) If the particle is initially at rest then from v = u + at, we get


1 2
And from Eqn. s ut at
2

qE
t
m

1 2 1 qE 2
at
t
2
2 m

E
+
+
+
+
+
+
+

+
+
+
+
+
+
+

we get s

v = at =

-q

+q

d
PD=V

v0

D
GyaanSankalp

Electric charges and fields


The motion is accelerated translatory with a t;
Here W KE

1 2 1 qE
mv m t
2
2 m

v t

and s t

also W = qEd = qV

(ii) If the particle is projected perpendicular to the field with an initial velocity v0
1
at ,
2
ux = v0 = constant and x = v0t

From Eqn. v = u + at and s = ut +

for motion along y-axis as uy = 0 and ay =

ux = v0 and ax = 0,

1 qE 2
qE
qE
, v y t and y t
m
2 m
m

2

So eliminating t between equation for x and y, we have y

qE x
qE 2
x

2m v0
2mv20

If particle is projected perpendicular to field the path is a parabola.


Example 13 :
When a 5nC test charge is placed at a certain point, it experiences a force of 2 104 N in the x-direction. What is the electric field

E at the point ?

Sol. E = F / q0 = [(2 104 N) i ] / (5 109 C) = (4 104 N/C) i )
Example 14 :
Four particles, each having a charge q, are placed on the four vertices of a regular pentagon. The distance of each corner from the
centre is a. Find the electric field at the centre of the pentangon.
Sol. Let the charges be placed at the vertices A, B, C and D of the pentagon ABCDE. If we
put a charge q at the corner E also, the field at O will be zero by symmetry. Thus, the
B
A
field at the centre due to the charges at A, B, C and D is equal and opposite to the field
due to the charge q at E alone.
q

The field at O due to the charge q at E is

4 0 a

Thus, the field at O due to the given system of charges is

along EO.

q
4 0 a 2

D
along OE.

Example 15 :
Two positive charges Q1 and Q2 are placed on a line as shown in figure. Determine the position of point O, where the net electric
field is zero.
Sol. Let position of P is at a distance x from Q1. Then the fields at P due to Q1 and Q2 are in opposite directions. They will add up to
give zero, only if their (electric field's) magnitude are equal. That is

kQ1
x

kQ 2
(R x) 2

R x

=
x

Q2
Q1

or

x=

The distance of point P from charge Q is

Q1

1 Q 2 / Q1

d=Rx=

Q2

P
R

R
1 Q1 / Q2

If two negative charges are placed on a line (instead of positive charges), then the position of point P where the net electric field
is zero, is again

x = R / 1 Q2 / Q1

10

GyaanSankalp

, d = R / 1

Q1 / Q 2 .

Electric charges and fields


Example 16 :
Calculate the electric field strength required to just support a water drop of mass 107 kg and having charge 1.6 1019 C.
Sol. Here, m = 107 kg, q = 1.6 1019 C
Step 1 : Let E be the electric field strength required to support the water drop.
Force acting on the water drop due to electric field E is
F = qE = 1.6 1019 E
Weight of drop acting downward,
W = mg = 107 9.8 newton.
Step 2 : Drop will be supported if F and W are equal and opposite.
i.e., 1.6 1019 E = 9.8 107 or

E=

9.8 10 7
1.6 10 19

= 6.125 1012 N C1.

Example 17 :
Two charges of + 10 C and + 40 C respectively are placed 12cm apart. Find the position of the point where electric field is zero.
Sol. Let P be the point at a distance x from the charge + 10 C where electric field due to two charges + 10 C and + 40 C is zero.
Electric field intensity due to q1 at P,

E1 =

Electric field intensity due to q2 at P,

E2 =

Clearly, field at P will be zero if


i.e.

q2
q1
1
1
2 =
(r

x)2
4 0
4 0 x

or

C = 10 106 C ;
6

Here, q1 = 10
40 10
10 10 6
=

2
(r x)2
x

q1
1
; along PB
4 0 x 2

q2
1
A.
2 ; along PA.
4 0 (r x)
E1 = E2
q1
x

q2
(r x)2

q2 = 40 C = 40 106 C

(r x)2 = 4x2 or (r x) = 2x

3x = r or x =

r
3

12
= 4.0 cm.
3
Thus electric field will be zero at a distance of 4.0 cm from the charge + 10C.
Example 18 :
Can a metal sphere of radius 1cm hold a charge of 1 coulomb.
Sol. Electric field at the surface of the sphere.

But r = 12cm (given)

E=

KQ
R

9 109 1

2 2

(1 10 )

= 9 1013

x=

V
m

This field is much greater than the dielectric strength of air (3 106 v/m), the air near the sphere will get ionised and charge will leak
out. Thus a sphere of radius 1 cm cannot hold a charge of 1 coulomb in air.

TRY IT YOURSELF
Q.1 Two charges of opposite nature having magnitude 10 C are 20 cm apart. The electric field at the centre of line joining these
charges will be(A) 9 x 106 N/C in the direction of positive charge
(B) 18 x 106 N/C in the direction of negative charge
(C) 18 x 106 N/C in the direction of positive charge
(D) 9 x 106 N/C in the direction of negative charge
Q.2 A point charge A of charge +4 C and another point charge B of charge 1 C are placed in air at a distance 1 meter apart. Then
the distance of the point on the line joining the charges and from the charge B, where the resultant electric field is zero, is- (in
metre)
A
(A) 2
(B) 1
(C) 05
(D) 15
Q.3 Four charges each +q, are placed at the four corners of a regular pentagon as shown
in the fig. The distance of each corner from the centre O is r. Then the electric field at
the center will beq
(A) 4 r towards OA
0

q
(B) 2 2 r towards OA
0

(C) Zero

q
(D) r towards OA
0

O
q

GyaanSankalp

11

Electric charges and fields


Q.4 A charge Q is divided into two parts such that charge on each part is q and Q/q = 2. The Coulombic force between the two charges
q and q when placed some distance apart
(A) is maximum irrespective of the medium in which they are placed
(B) is minimum
(C) is equal in magnitude for both opposite in direction
(D) is depenent on the medium in which charges are placed
Q.5 Two free point charges +4Q and +Q are placed at a distance r. A third charge q is so placed such that all the three are in equilibrium
(A) q is placed at a distance r/3 from 4Q
(B) q is placed at a distance r/3 from Q
(C) q = 4Q/9
(D) q = 4Q/9
Q.6 A charge q = 1 C is placed at point (1m, 2m, 4m). Find the electric field at point P (0m, 4m, 3m).
Q.7 A copper ball of diameter d is immersed in an oil of density 0. There is a homogeneous electric field E directed vertically upwards
such that the copper ball is suspended in the oil. Density of copper is C. The charge on the ball is
d3 (c 0 )g
d3 (c 0 )g
d3 (c 0 )g
d3 (c 0 )g
(B)
(C)
(D)
3E
2E
6E
12E
Q.8 A charge 109 coulomb is located at origin is free space and another charge Q at (2, 0, 0). If the x-component of the electric field
at (3, 1, 1) is zero, calculate the value of Q. Is the y-component zero at (3, 1, 1) ?
Q.9 A charged particle of mass m = 2 kg and charge 1 C is thrown from a horizontal ground at an angle = 45 with speed 10 m/s. In
space a horizontal electric field E = 2 107 N/C exists. The range of the projectile is
(A)20m
(B) 60m
(C) 200m
(D) 180m
Q.10 In the fig. distance of the point from A where the electric field is zero isA
B
(A) 20 cm
(B) 10 cm
(C) 33 cm
(D) None of these
80cm
10 C
20 C

(A)

ANSWERS
(1) (B)

(2) (B)

(3) (A)

(4) (A)

N
(6) E (38.42i 250.52j 38.42k)
C

(7) (C)

3
(8)
11

(9) (A)

(10) (C)

(5) (BD)
3/2

3 109 C

ELECTRIC FIELD DUE TO CONTINUOUS CHARGE DISTRIBUTION


The evaluate the electric field of a continuous charge distribution, the following procedure is used. First, we divide the charge
distribution into small elements each of which contains a small charge q. Next, we use Coulomb's law to calculate the electric field
due to one of these elements at a desired point. Finally, we evaluate the total field at a point due to the charge distribution by
summing the contributions of all the charge elements (that is, by applying the superposition principle.)
Let us consider some cases
Case 1 : The electric field of a uniform ring of charge.
A ring of radius a has a uniform positive charge per unit length, with a total charge Q. To find the electric field along the axis of the
ring at a point P lying a distance x from the center of the ring follow the procedure.
The magnitude of the electric field at P due to the segment of charge q is E = k
1

This field has an x component Ex = E cos along the axis of


the ring and a component E perpendicular to the axis. But as
we see in figure, the resultant field at P must lie along the x axis
since the perpendicular components sum up to zero. That is, the
perpendicular component of any element is canceled by the perpendicular component of an element on the opposite side of the
ring. Since r = (x2 + a2)1/2 and cos = x/r, we find that

12

GyaanSankalp

r2

E2

E1

kx
q x
q
Ex = E cos = k 2 r 2
r
(x a 2 )3/2

In this case, all segments of the ring give the same contribution
to the field at P since they are all equidistant from this point.
Thus, we can easily sum over all segments to get the total field

Electric charges and fields


at P :

Ex =

kx
2

2 3/2

(x a )

q =

kx
2

(x a 2 ) 3/2

This result shows that the field is zero at x = 0.


At large distances from the ring (x > > a) the electric field along the axis approaches that of a point charge of magnitude Q.
Case 2 : The electric field of a uniformly charged disk :
A disk of radius R has a uniform charge per unit area . To find the electric field along the axis of the disk, a distance x from its center
dq
follow the procedure.
Consider the disk as a set of concentric rings. We can then make
use result of case 1,which gives the field of a given ring of radius
r, and sum up contributions of all rings making up the disk. By
symmetry, the field on an axial point must be parallel to this axis.
The ring of radius r and width dr has an area equal to 2r dr. The
r
charge dq on this ring is equal to the area of the ring multiplied
by the charge per unit area, or dq = 2r dr. Using this result in
x
equation (with a replaced by r) gives for the field due to the ring
the expression.
dE =

kx
2

(x a 2 ) 3/2

(2r dr)

To get the total field at P, we integrate this expression over the limits
r = 0 to r = R, noting that x is a constant, which gives
E = kx

2r dr

R
0

(x 2 r 2 )3/ 2

= kx

(x2 + r2)3/2 d(r2) = kx

R
x

x
(x 2 r 2 ) 1/2

= kx | x | (x 2 R 2 )1/ 2
1/ 2

The result is valid for all values of x.


The field close to the disc along an axial point can also be obtained from equation by letting x 0
This gives

E= 2 k =

2 0

where 0 is the permittivity of free space, the same result is obtained for the field of a uniformly charged infinite sheet.

Case 3 : E due to an infinite plane of charge


The field of an infinite plane of charge can be obtained from field by disc by either letting R go to infinity or letting x go to zero.
Then
Ex = 2k, x > 0
Thus, the field due to an infinite-plane charge distribution is uniform; that is, the field does not depend on x. On the other side of
the infinite plane, for negative values of x, the field points in the negative x direction, so Ex = 2 k , x < 0
Case 4 : Spherical distribution of charge
(a) Conducting sphere (Hollow, solid) (b) Non-conducting sphere (Hollow, solid)
Case (a) Charge on surface.
Case (b) Volume distribution of charge.
(a) : Hollow/solid conductor or hollow non-conductor
+ + + +
Q+
Imagine a sphere passing through desired point (point where E is
+
+
+
to be calculated), calculate charge inside it and assume it
+
to be concentrated at centre and use point charge formula.
+
r R
+
Inside sphere r < R
+
+
E = 0 (No charge inside imagined sphere)
+

outside r > R
E=

+
+
+
+

KQ
r2

surface r = R , E =

+Q+ + +

R
+ + ++

+
+
+
+

KQ
R2
GyaanSankalp

13

Electric charges and fields


E
E 1/r 2

KQ/R2

Graphically

r=R

(b) Inside r < R


Uniform volume distribution : charge inside volume

Q +

KQ
Qr 3
KQ1
4 3
Q 4 3
r
r = Q' or Q' = 3 , E = 2 = 3 r
4
3
3
R
R
r
R 3
3

E
r
3 0

Q
KQ

Outside, E =
Surface E =

r3

KQ
R3

+ +
+
+ +
+
r+
+
+
+
+
+
+ R
+
+
+

++
+ + ++
++
r+
+ + ++
+ R +
++

r>R

,
E
r

Graphically

r=R

1 /r 2

Electric field intensities due to various charge distributions are given in table.
Formula

Name/Type
Point charge

Kq
Kq
.r 3 r
| r |2
r

Infinitely long
line charge

2K r
r
2 0 r
r

2k
, E x Kr , E y kr
r

Semi-infinite

Ex

Finite change of
charge
+
+
+
+
+
+
+
+
+

14

P
x
E

GyaanSankalp

Ey

K
[sin sin ]
r

K
[cos cos ]
r

If =
E

E|| 0, E

2 0 r

Particular
q is source charge.
is vector drawn from source charge
r
to the test point.
Electric field is nonuniform, radially
outwards due to + charges & inwards
due to charges.
is a linear charge density
(assumed uniform)
r is perpendicular distance of point
from line charge.
r is radial unit vector drawn from the
charge to test point.
At a point above the end of wire at an
angle 45
Where is the linear charge density

Graph

Electric charges and fields


Infinite nonconducting
thin sheet

is surface charge density


(assumed uniform)
n is unit normal vector..
Electric field intensity is
independent of distance.

n
2 0

r
Uniformly charged
ring
E

Infinitely large
charged conducting
sheet

Uniformly charged hollow


conducting/
nonconducting/
solid conducting
sphere

KQx
(R 2 x 2 )3/ 2

Q is total charge of the ring.


x = distance of point on the axis
from centre of the ring.
Electric field is always along the axis.

E centre 0

Maximum at x R / 2

n
0

is the surface charge.


n is unit normal vector
perpendicular is the surface.
Electric field intensity is
independent of distance.

R is radius of the sphere.


is a vector drawn from centre
r
of sphere to the point.
Sphere acts like a point charge,
placed at centre for points
outside the sphere.

E is always along radial direction.

(i) for r R

kQ
E 2 r
|r|
(ii) for r < R

E0

Q is total charge ( 4 R 2 )
( = surface charge density)
Uniformly charged
solid nonconducting
sphere (insulating
material)

is a vector drawn from centre


r
of sphere to the point.
Sphere acts like a point charge, placed
at centre for points outside the sphere.

E is always along radial direction.

(i) for r R

kQ
E 2 r
|r|
(ii) for r R
KQr r
E 3
3 0
R

Uniformly charged cylinder with a charge density


(R = radius of cylinder)

r
for r < R, E in 2
0

for r > R, E
Uniformly charged cylindrical shell with surface
charge density is

4
3
Q is total charge (. 4 R )
3
( = volume charge density)
Inside the sphere E r
Outside the sphere E 1/r

R 2
2 0 r

E 1/r

for r < R, Ein = 0,


r
for r > R, E r
0
GyaanSankalp

15

Electric charges and fields


Example 19 :
Electric charge is uniformly distributed around a semicircle of radius a, with total charge Q. What is the electric field at the centre
of curvature ?
Sol. Consider a small segment of angular width d, located at an angle from the x-axis.
The length of the segment is ds = ad. The charge on the segment is dQ Q

ad Q
d
a

1 dQ
1
Q

d
4 0 a 2 4 0 a 2

The magnitude of the electric field at P is given by

dE

This electric field has y-component

dE y dE sin

Q
2

4 0 a 2

sin d

The x-component of the field from the right hand half of the ring cancels with that of the left-hand half of the ring. The resultant
electric field is thus in the y-direction, and is given by adding up the dEy from each segment in the ring. This is done by integrating
from to rad :

E y dE y
0

Ey

4 0 a

4 0 a 2

sin d

Q
2

Q
2

sin d 4 2 a 2 cos
0

; Ey

Q
2

4 0 a

[( 1) (1)]0 ;

Ey

Q
2

2 0 a 2

Example 20 :
A thin non-conducting ring of radius R has linear charge density 0 cos , where 0 is a constant, is the azimuthal angle.
Find the magnitude of the electric field strength at the centre of the ring.
Sol. The situation is shown in figure.
The half ring on the right hand side will be positive while on the half left side
will be negative. The reason being that cos for first and fourth quadrants is
positive while for 2nd and 3rd quadrants is negative.
Consider a small element dx of the ring. Here dx = R cos
Charge on small element dq = = dx = 0 cos (R d)
R 0 cos d
1
dE 4
R2
0

Electric field along x-axis due to this element


dE x dE cos
=

0 1 cos 2
R0 cos d
1
0

cos =
(cos2 d ) =
d
2
4 0 R
2
4 0
4 0 R
R

Electric field due to positive part along x-axis,


16

GyaanSankalp

E1

0
4 0 R

/ 2

1 cos 2

d
2

/ 2

Electric charges and fields


or E1

0
4 0 R 2 8 0 R

Similarly, the electric field due to negative charge along x-axis


E2

0
8 0 R

0
0
0
Enet = E1 + E2 = 8 R 8 R 4 R
0
0
0

TRY IT YOURSELF
Q.1 A spherical volume contains a uniformly distributed charge of density . The electric field inside the sphere at a distance r from
center is

(A) 3 r
0

(B) 4 r
0

(C) r
0

1
(d) 4 r
0

Q.2 A point charge 50C is located in the XY plane at the point of position vector r0 2i 3j . What is the electric field at the point

of position vector r 8i 5j
(A) 1200 V/m
(B) 0.04 V/m
(C) 900 V/m
(D) 4500 V/m
Q.3 A solid metallic sphere has a charge +3Q concentric with this sphere is a conducting spherical shell having charge Q. The radius
of the sphere is a and that of the spherical shell is b (>a). What is the electric field at a distance r (a < r < b) from the centre
1 Q
(A) 4 r
0

1 3Q
(B) 4 r
0

1 3Q
1 Q
(C) 4 2
(D) 4 2
0 r
0 r
Q.4 The maximum electric field intensity on the axis of a uniformly charged ring of charge q and radius R will be
1
q
1 2q
1
2q
1
3q
(A) 4
(B) 4
(C) 4
(D) 4
2
2
2
2
0 3 3R
0 3R
0 3 3R
0 2 2R
Q.5 Two conducting plates X and Y, each having large surface area A (on one side) are placed parallel to each other. The plate X is
given a charge Q where the other is neutral. The electric field at a point in between the plates is given by

(A)

Q
(B) 2A towards left
0

Q
2A

(C)

Q
towards right
2A 0

Q
(D) 2 towards right
0

Q.6 Two infinitely long parallel wires having linear charge densities 1 and 2 respectively are placed at a distance R meter. The force

1
per length on either wire will be K 4

1 2
21 2
(C) K 2
R
R
R
Q.7 A wheel having mass m has charges + q and q on diametrically opposite
points. It remains in equilibrium on a rough inclined plane in the presence of
uniform vertical electric field E =
(A) K

21 2
2

(B) K

(A)

mg
q

(B)

(C)

mg tan
2q

(D) none

1 2
R

(D) K

+q
E
q

mg
2q

ANSWERS
(1) (A)
(5) (C)

(2) (D)
(6) (B)

(3) (C)
(7) (B)

(4) (C)

GyaanSankalp

17

Electric charges and fields


ELECTRIC FIELD LINES
Electric lines of forces :
(i) The concept of electric field was introduced by Michael Faraday.
The magnitude of electric field strength at any point is measured by the number of electric line of force passing per unit small area
around that point normally and the direction of field at any point is given by the tangent to the line of force at the point.
(ii) An electric line of force is that imaginary smooth curve drawn in an electric field along which a free isolated unit positive (initially
at rest) charge moves.

E at any point on a line of force.


Properties :
(1) The lines of force diverge out radially from a +ve charge
and converge at a ve charge. More correctly the lines
of force are always directed from higher to lower potential.

(2) The tangent drawn at any point on line of force gives the direction of force acting on a positive charge placed at that point.
(3) Two lines of force never intersect. If they are assumed to intersect. There will be two directions of electric field at the point of
intersection : which is impossible.
(4) These lines have a tendency to contract in tension like a stretched
elastic strong. This actually explains attraction between opposite
charges.

Attraction

(5) These lines have a tendency to separate from each other in the direction
perpendicular to their length. This explains repulsion between like
charges.

Repulsion

(6) The no. of lines originating or terminating on a charge is proportional to the magnitude of charge. In rationalised MKS system
(1/ 0 ) electric lines are associated with unit charge. So if a body encloses a charge q. Total line of force associated with it (called
q
flux) will be .
0

A
B
Total lines of force may be fractional as lines of force are imaginary.
qA>qB
Lines of force ends or strarts normally on the surface of a conductor.
If there is no electric field there will be no lines of force.
Lines of force per unit area normal to the area at a point represents magnitude of intensity, crowded lines represent strong field
while distant lines represent weak field.
(11) Electric lines of force differ from magnetic lines of force.
(a) Electric lines of force never form closed loop while magnetic lines are always closed or extended to infinity.

(7)
(8)
(9)
(10)

Electric line of force


(A)

18

GyaanSankalp

Magnetic line of force


(B)

Electric charges and fields


(b) Electric lines of force always emerge or terminate normally on the surface of charged conductor, while magnetic lines emerge or
terminate on the surface of a magnetic material at any angle.
(c) Electric lines of force do not exist inside a conductor but magnetic
lines of force may exist inside magnetic material.
Lines of force do not exist inside a conductor (as field inside a
conductor is zero) the plates is as shown. (Electrostatic shielding)
12. Neutral point : Where electric field intensity is zero (test charge does not experience any force)
13. The number of lines leaving a positive charge or entering a negative charge is proportional to the magnitude of the charge. This
means, for example that if 100 lines are drawn leaving a + 4C charge then 75 lines would have to end on a 3C charge.
ELECTRIC FLUX ( ) :
If the lines of force pass through a surface then the surface is said to have flux linked with it. Mathematically it can be formulated
as follows :
The flux linked with small area element on the surface of the body :

d =


E.d s

Where d s is the area vector of the small area element. The area vector of a closed surface is always in the direction of outward
drawn normal. The total flux linked with whole of the body,

=
E.ds total flux linked with closed surface, where is referred to closed integral done for a closed surface.
(i)
(ii)
(iii)
(iv)

Electric flux is a scalar quantity


Units (V - m) or (N - m2/Cb),Dimensions : [M1 L3 T3 A1]
The value of '' does not depend upon the distribution of charges and the distance between them inside the closed surface.
The value of is zero in the following circumstances :
(a) If a dipole is enclosed by a closed surface.
(b) Magnitude of +ve and ve charges are equal inside a closed surface.
(c) If no charge is enclosed by a closed surface.
(d) In coming flux (ve) = out going flux (+ ve).

GAUSS'S LAW
1
The total flux linked with a closed surface is times the charge enclosed by the closed surface (Gaussian surface).
0

i.e.

q

E. ds =
0

Law is valid for symmetrical charge distribution and for all vector fields obeying inverse square law.
Gaussian surface :
(a) Imaginary surface
(b) Is spherical for a point charge, conducting and non-conducting spheres.
(c) Is cylindrical for infinite sheet of charge conducting charge surfaces, infinite line of charges, charged cylindrical conductors,
etc. For finite charge distribution use Coulomb's law.
For infinite charge distribution use Gauss theorem

q net
E. ds =
0

Application : (1) To Calculate flux


(2) To calculate Electric field Intensity
Study following cases to learn application properly
Observe flux through common geometrical figures

(i) out = in = R2E

(ii) in = out = Ea2 ;

total = 0.

GyaanSankalp

19

Electric charges and fields

q
q
(iv) T = , hemisphere = 2
0
0

(iii)T = 0

(dotted part shows imaginary part to enclosed the charge completely)

q
q
(v) T = , cyl. = 2 .
0
0

q
cube = 2
0

T
= q/.
= q/2.
= q/8.
= q/40.

Charge position
Cube centre
Face centre
At corner
At centre of edge

1
T = 8
0

q
(vi) T =
0

qi

i = 1, 2, ............ 8.

Electric field due to a line charge :


Consider an infinite line which has a linear charge density . Using Gaussss
law, let us find the electric field at a distance r from the line charge.
The cylindrical symmetry tells us that the field strength will be the same at all
points at a fixed distance r from the line. Thus, the field lines are directed
radially outwards, perpendicular to the line charge.
The appropriate choice of Gaussian surface is a cylinder of radius r and

S2

length L. On the flat end faces, S 2 and S 3 , E is perpendicular dS , which means

S3

flux is zero on them. On the curved surface S1 , E is parallel dS , so that E, dS EdS .


The charge enclosed by the cylinder is Q = L.
Applying Gausss law to the curved surface, we have

E dS E( 2 rL)

L
0

or

E ds cos

I circular
surface

20

GyaanSankalp

II circular
surface

E
r
S1

E
dS

E
dS

Gaussian

+
++
Surface
+ ++
+
+
P
+
+ + ++
r
+
+
Plane sheet
of charge

E .dS qin / 0
E ds cos

dS

2 0 r

Electric field due to an infinite plane thin sheet of charge :


To find electric field due to the plane sheet of charge at any point P distant r
from it, choose a cylinder of area of cross-section A through the point P as the
Gaussian surface. The flux due to the electric field of the plane sheet of charge
passes only through the two circular caps of the cylinder. Let surface charge
density =
According to gauss law

+
+
+
+
+
+
+
+
+
+
+
+
+
+
+
+
+
+
+
+

cylindrical
surface

E ds cos

A
0

or

EA + EA + 0 =

A
2 0

or

E = 2
0

Electric charges and fields


Electric field intensity due to uniformly charged spherical shell :
We consider a thin shell of radius R carrying a charge Q on its surface
(i) at a point P0 outside the shell (r > R)

E0 .ds

According to gauss law ,

S1

E0 =

4 0 r 2

Q
Q
or E0 (4r2) =
0
0

Pin

ds

Ps

S1
S

S2 O R

R2

= 0

P0

r2

total charge
Q
where the surface charge density = surface area =
4R 2
The electric field at any point outside the shell is same as if the entire charge is concentrated at centre of shell.

ES =

(ii) at a point Ps on surface of shell (r = R)

Q
4 0 r 2

(iii) at a point Pin inside the shell (r < R)


According to gauss law

E .ds

qin
=
0

Q/4

2
0R

E
1

E=0

E=0

r2

r2

O r<R r=R
r>R
As enclosed charge qin = 0 , So Ein = 0
distance from centre (r)
The electric field inside the spherical shell is always zero.
Electric field intensity due to a spherical uniformly charge distribution :
We consider a spherical uniformly charge distribution of radius R in which total charge Q is uniformly distributed throughout the
S2

3Q
Q
total charge
=
=
4
4R 3
total volume
R 3
3
(i) at a point P0 outside the sphere (r > R)
volume. The charge density =

or E0 =

4 0 r 2

+ R

+
+

Pin

+
+

Ps P0

r
+ +

O
+

3
R
2
=
3 0 r

+
+

Q
Q
according to gauss law E 0 .ds =
or E0 (4r2) =
0
0

ds

+
+

(ii) at a point Ps on surface of sphere (r = R)


Q

Es =

4 0 R

R
3 0

(iii) at a point Pin inside the sphere (r < R)


According to gauss law
4 3
Qr 3
qin
1
=
=
.
r
=
E
.ds
in

3
0
0
0 R 3

Ein(4r2)

Qr 3
0 R 3

or Ein =

Qr
4 0 R 3

r2

=
r (Ein r)
3 0

r<R

r=R

1
r2
r>R

MP

ELECTROSTATIC PRESSURE
To find force on a charged conductor (due to repulsion of like charge) imagine
a small part PR to be cut and just separated from the rest of the conductor
MLN. The field in the cavity due to the rest of the conductor is E2, while field
due to small part is E1. Then

R
N

L
GyaanSankalp

21

Electric charges and fields


Inside the conductor :
E = E1 E2 = 0 or
Outside the conductor :

E1 = E2

E E1 E 2 / 0
E1 E 2

Thus

2 0

To find force, imagine charged part PR (having charge dA placed in the cavity MN having field E2. Thus force
dF (dA) E 2

dF

or

2
dA
2 0

The force per unit area or electric pressure is

dF 2

dA 2 0
The force is always outwards as ( )2 is positive i.e. whether charged positively or negatively, this force will try to expand the
charged body.
A soab bubble or rubber balloon expands on given charge to it. (charge of any kind + or )
Energy associated per unit volume of electric field of intensity E is defined as energy density.
u=

0E 2
dw

=
=
J/m3
2
dv
2 0

U=

u . dv =

0
2

E2 dv

v is the volume of electric field.

Equilibrium of charged liquid surfaces :


Soap Bubble :
Pressures (forces) acts on a charged soap bubble, due to
(i) Surface tension of a soap bubble PT (inward)
(ii) Air out side the bubble p0 (inward)
(iii) Electric charges (electrostatic pressure) Pe (outward)
(iv) Air inside the soap bubble Pi (outward)
Hence, in state of equilibrium
inward pressure = outward pressure
PT + P0 = Pi + Pe
Excess pressure
(Pex.) = Pi P0 = PT Pe
But PT =

4T

, Pe =
r
2 0

Pex. =

4T

2 0
r

4T
If Pi = P0, then
=
2 0
r
Example 21 :

There is a solid sphere of radius R having volume charge density 0 1 , where 0 is any constant and r is the distance
R
from the centre of sphere. Find electric intensity E inside and outside the sphere.
22

GyaanSankalp

Electric charges and fields


Sol. (i) Inside : When r < R, electric flux through small area,

2
d E.dS E dS
E dS E dS E 4r
But according to Gauss's law

qenclosed
0

Charge q contained between radius x to x + dx.

dq 4x 20 1 dx
R
Charge enclosed inside the gaussian surface,
r

q enclosed

x3 x 4
x3
40 x 2 dx 40 = 40
R
R
3
0
0

But E 4r 2

40 r3 r 4

0 3 4R

0 r 1 r

0 3 4R
(ii) Outside : When r R, then

x3 x 4
q enclosed 4 0
40
3 4R 0

r3 r 4
3 4R

0 r
3r
1

3 0
4R , when r R

R
R

r3 r 4

3 4R 0

0 R 3

12 0 r 2

Example 22 :
A charge of 4 108 C is distributed uniformly on the surface of a sphere of radius 1cm. It is covered by a concentric, hollow
conducting sphere of radius 5 cm.
(a) Find the electric field at a point 2 cm. away from the center
(b) A charge of 6 108 C is placed on the hollow sphere.
Find the charge on the outer surface of the hollow sphere.
Sol. (a) Let us consider figure (a). Suppose, we have to find
field at the point P. Draw a concentric spherical through
P. All the points on this surface are equivalent and by
symmetry, the field at all these points will be equal in
P
magnitude and radial in direction.
The flux through this surface

2
=
E.dS E.dS E dS 4x E
(a)
(b)
where x = 2 cm = 2 102 m
From Gausss law, this flux is equal to the charge q contained inside the surface divided by 0.
Thus, 4x 2 E q
0
or E

q
4 0 x 2

(9 109 Nm 2 / C 2 )

4 108 C
4 104 m 2

9 105 N / C

(b) See figure (b). Take a Gaussian surface through the material of the hollow sphere. As the electric field in a conducting material

GyaanSankalp

23

Electric charges and fields


is zero, the flux

E.dS through this enclosed must be zero. Hence, the charge on the inner surface of the hollow sphere is

4 108 C. But the total charge given this hollow sphere is 6 108 C. Hence, the charge on the outer surface will be
10 108 C.
Example 23 :
A gaussian surface encloses an object with a net charge of +2.0 C and there are 6 lines leaving the surface. Some charge is added
to the object and now there are 18 lines entering the surface. How much charge was added ?
Sol. Since there are 6 lines when there is +2.0 C, therefore a charge of +1.0 C is equivalent to 3 lines. After charge is added, there are 18
lines entering.
18 lines
So the net charge is now 3 lines/coulomb 6.0 C

Therefore, the charge added was Q Qf Qi 6.0 C 2.0 C 8.0C


Example 24 :
The length of each side of a cubical closed surface is . If charge q is situated on one of the vertices of the cube, then find the flux
passing through each face of the cube.
Sol. The charge contributes only (q/8)th to this cube. Further, three faces of the cube meet at one corner which are parallel to the
charge. From Gauss theorem
one face

(q / 8)
q

3 0
24 0


And from three faces meeting at point where charge is placed = 0 as E A

TRY IT YOURSELF
Q.1 A charge is placed at the centre of a cube with side L. The electric flux linked with cubical surface is
(A) (Q / 6L2 0 )
(B) (Q / L2 0 )
(C) (Q / 0 )
(D) zero
Q.2 A charge Q is situated at the centre of a cube. The electric flux through one of the faces of the cube is
(A) (Q / 0 )
(B) (Q / 2 0 )
(C) (Q / 4 0 )
(D) (Q / 6 0 )
Q.3 A charge q is placed at the centre of the open end of a cylindrical vessel. The flux of the electric field through the surface of the
vessel is
(A) zero
(B) (q / 0 )
(C) (q / 2 0 )
Q.4 A hemispherical surface of radius R is placed with its cross-section
perpendicular to a uniform electric field E as shown in fig. flux linked with its
curved surface is
(A) zero

(D) (2q / 0 )
E

(B) 2R 2 E
R

(C) R 2 E
(D) (E / 2 0 )
Q.5 The application of Gauss's theorem gives rise to an easy evolution of electric field in the case of
(A) A charged body of any geometrical configuration
(B) A charged body of regular geometrical configuration
(C) Revolving charged bodies
(D) Charges forming dipoles
Q.6 Three charges q1 = 1c, q2 = 2c and q3 = 3c and four surface S1, S2,
S3 and S4 are shown. The flux emerging through surface S2 in N-M2/C
is
s3
q3
q1
(A) 36 103
(B) 36 103
q2
(C) 36 109
(D) 36 109
s2
s1
Q.7

A cubical box of side 1m is immersed a uniform electric field of strength 104N/C. The flux through

s4
the cube is

(A) 104
(B) 6 104
(C) 2 104
(D) zero
Q.8 If an insulated non-conducting sphere of radius R has charge density . The electric field at a distance r from the centre of sphere
(r < R) will be
r
(A) 3
0

24

GyaanSankalp

R
(B) 3
0

r
(C)
0

R
(D)
0

Electric charges and fields


Q.9 The inward and outward electric flux from a closed surface are respectively 8 10 and 4 103 units. Then the net charge inside
the closed surface is

4 103
(A)
coulomb
0

(B) 4 103

4 103
0 coulomb (C)
coluomb
0

(D) 4 103 0 coulomb

ANSWERS
(1) (C)
(6) (B)

(2) (D)
(7) (D)

(3) (C)
(8) (A)

(4) (C)
(9) (B)

(5) (B)

ELECTRIC DIPOLE
In some molecules, the centre of +ve and -ve charge do not coincide. This results in the formation of electric dipole. Atom is nonpolar because in it the centre of +ve and -ve charges coincide. Polarity can be induced in an atom by the application of electric
field. Hence it can be called as induced dipole.
An electric dipole is a system formed by two equal and opposite charges placed at a short distance apart. Product of one of the

two charges and the distance between them is called "electric dipole moment" p .

p q 2
(1) It is a vector quantity, directed from - ve to + ve charge.
(2) Dimension [L T A], unit Cb x mt.

(3) Practical unit is Debye p of two equal and opposite point charges each having charge 1010 frankline and separation of 1 .
i.e. 1D = 1010 x 1010 = 1020 Fr x mt. =

10 20
3 x 109

Cb x mt.

1D 3.3 x 1030 Cb x mt.

Electric field at an axial point :


Electric field at P due to negative charge

E1

1
q

( r)
4 0 d 2
r
2

r
p O
-q

P
+q

1
q

( r)
4 0 d 2
r
2

Ea E1 E2

Electric field at P due to positive charge E 2

Total electric field at axial point P is

Ea =

1
1
2.r.d

q
q
2pr
r or E

r =
r
=

2
2
2
2
2
a
(r d / 2)
4 0 (r d / 4)
4 0 (r d / 2)
4 0 (r 2 d 2 / 4) 2

1
2p

r
When r >> d Ea = 4
3 , The axial field is parallel to dipole moment
0 r
Electric field at an equatorial point :

E 2sin

E2
E 2cos

q
1
Electric field at P due to negative charge E1 =
4 0 (r 2 d 2 / 4)

E 1cos
E1

1
Electric field at P due to positive charge E 2
4 0

2
r d / 4

Fields E1 and E2 are equal in magnitude


Resolving E1 and E2 into two components one along OP and other perpendicular to OP
We find E1sin = E2 sin
Total field E = E1cos + E2cos = 2E1cos = 2E2cos

r2

d2 / 4

-q

r2

d2 / 4

E 1sin

dO

GyaanSankalp

25

Electric charges and fields


1
q
E = 2 4
2
2
0 (r d / 4)

d/2
2

r d /4

q.d
2

4 0 (r d / 4)

3/2

4 0 (r d 2 / 4)3/2

1.

p
1

E = 4 0 r 3 r i.e. field at equatorial point is antiparallel to dipole moment.


From this it is clear that :
Intensity due to a dipole varies as (1/r) and can never be zero unless r or p 0 .

2.

2
E will be maximum when cos max 1 , i.e., 0 , for end on, axial or tan A position E is maximum and is,

If r >> d

E max

1 2p
4 0 r 3

3.

E will be minimum when cos 2 min 0 , i.e., 90 , i.e., for broad on, equatorial or tan B position E is minimum and is,

4.
5.
6.

1 p
4 0 r3
The electric field at axial point is parallel to dipole moment vector.
The electric field at equatorial point is antiparallel to dipole moment vector.
The ratio of field at axial point to field at equatorial point is Ea : E = 2 : 1.
Electric field at an arbitrary point :
We resolve dipole moment p in two components one along r and another perpendicular to r.
E min

The radial component of electric field Er =

1 2p cos
4 0
r3

psin
1
The magnitude of resultant field is E =
r3
4 0
The magnitude of resultant field is E =

Er 2 E2

1 3cos 2

+q

r
pcos

p
B -q

E 1
The direction of resultant field is tan = E 2 tan
r
1
p
so Ea = 4 3
0 r

Case I at axial point =0

1 2p
1 3cos2 0 = 4 0 r 3

1
1
p
p
Case II at equatorial point = /2 so E = 4 3 1 3cos 2 / 2 = 4 3
0 r
0 r
Dipole in a field :
(1) When an electric dipole is placed in an uniform electric field.
A torque acts on it which subjects the dipole to rotatory motion

F net = [q E + (-q E )] = 0

Torque = q E x 2 sin = P x E
There is no net force acting on the dipole in a uniform electric field.
(2) Work : Work done in rotating an electric dipole from to (uniform field)

dw = dw =

z z
dw =

d.

W =

pE sin d= pE (cos cos )

e.g. W180 = pE (1 (1)] = 2pE


W 90 = pE (1 0) = pE
26

GyaanSankalp

y
A
ps
in

p
1
=
3
r
4 0

Er

+q
F=qE
2l
2l sin
q

Electric charges and fields


If a dipole is rotated from field direction (= 0) to , then W = pE (1 cos )

= min = 0
= max. = pE
= min = 0
W = min = 0
W = pE
W = max. = 2pE
(3) Electrostatic potential energy :
In case of dipole (in uniform field) potential energy of dipole is defined as work done in rotating a dipole from direction to the
field to the given direction i.e.
U = w0 w90 = pE (1 cos ) pE = pE cos .

U = P.E

E is a conservative field so what work done in rotating a dipole from 1to 2is just equal to change in electrostatic P.E.
W1 2 = U U = pE (cos 1 cos 2 )
2


(4) If = 0, i.e. P || E , = 0 and U = pE,
Dipole is in the minimum potential energy state and no torque acting on it and hence it is in the stable equilibrium state.

For = 180, i.e. P and E are in opposite direction then = 0 but U = pE which is max. potential energy state. Although it is in
equilibrium, but it is not a stable state and a slight displacement can disturb it.
(5) If dipole is placed in a non-uniform electric field, it preforms rotatory as well as translatory motion, because now a net force also
acts on the dipole along with the torque.
+2

2 F'
In Uniform electric field, Total force = 0, Torque may or may not be zero. ( ( 0 if 0 )
F
In Non-uniform electric field, Total force 0, Torque may or may not be zero.
Q
For situation shown in figure, Torque = 0 (Force along same axis)
(6) Angular SHM :
When a dipole is suspended in uniform field, it will align itself parallel to the field. Now if it is given a small angular displacement
about its angular position, the restoring couple will be
= pE sin . if is small sin .
= pE
(Angular SHM).
for balanced condition :
deflecting = restoring

pE
I = pE = = 2 =
I
T=

2
= 2

I
pE

pE
I

[I moment of inertia]

Example 25 :
An electric dipole consists of charges 2.0 108 C separated by distance of
2mm. It is placed near a long line charge of density 4.0 104 cm1 as shown in
figure, such that the negative charge is at a distance of 2 cm from the line
charge. Calculate the force acting on the dipole.
Sol. We know that electric field intensity at a distance r from the line charge of
density is given by E

2 0 r

2cm

+q
2mm

Line charge

So, the field intensity on negative charge is given by E1


Force on negative charge,
F1 = qE1 = (2 108) (3.6 108)
This is directed towards line charge.

4.0 104
(2 9 109 ) = 3.6 108 N/C
0.02

F1 = 7.2 N

GyaanSankalp

27

Electric charges and fields


Similarly, field intensity E2 on positive charge

E2

4.0 104
(2 9 109 ) = 3.27 108 N/C
0.022

Force on positive charge, F2 = qE2.


F2 = (2 108) (3.27 108) = 6.54 N (away)
So, net force F on dipole, F = F1 F2 = (7.2 6.54) N = 0.66 N
The force is towards the line charge.
Example 26 :
Two points masses, m each carrying charge q nad +q are attached to the ends of a massless rigid non-conducting rod of length
. The arrangement is placed in a uniform electric field E such that the rod makes a small angle = 5 with the field direction. Show
that the minimum time needed by the rod to align itself along the field (after it is set free) is t

m
2 2qE

Sol. The situation is shown in figure.


The torque on rod AB is given by
qE( sin ) qE
The moment of inertia of the rod AB about O is given by
2

m


I m m
2
2
2

+q B

qE

We know that, I or
qE
2qE
2
(m2 / 2) m

where

2qE
m

qE

As acceleration is directly proportional to , hence the motion of rod is S.H.M. The time period T is given by
T

m
2
2

2qE

The rod will become parallel to E in a time


t

m m
T 2

4
4
2qE 2 2qE

TRY IT YOURSELF
Q.1 When a test charge is brought from infinity along the perpendicular bisector of the electric dipole the work done is
(A) Positive
(B) Negative
(C) Zero
(D) None of the above
Q.2 An electric dipole has charges +q and q at a separation r. At distance d >>r along the axis of the dipole, the field is proportional
to
Q.3
Q.4

Q.5

Q.6
Q.7

28

(A) q / d 2
(B) qr / d 2
(C) q / d 3
(D) qr / d 3
In case of a dipole field
(A) Intensity can be zero
(B) Potential can be zero (C) Both can be zero
(D) None can be zero
When an electric dipole is placed in a uniform electric field a couple acts on it. The moment of couple will be maximum when the
dipole is placed
(A) along the direction of the field
(B) perpendicular to the direction of the field
(C) against the direction of the field
(D) inclined at an angle of 45 to the direction of the field
The electric intensity due to a dipole of length 10cm. and having a charge of 500C, at a point on the axis 20cm. from one of the
charges in air is
(A) 9.28 107 N/C
(B) 20.5 107 N/C
(C) 6.25 107 N/C
(D) 13.1 1011 N/C
An electric dipole placed in a non-uniform electric field experiences
(A) A force but not a torque (B) A torque but not a force (C) A force and a torque (D) Neither a force nor a torque
An electric dipole consists of two opposite charges each of magnitude 1.0 C separated by a distance of 2.0cm. The dipole is
placed in an external field of 1.0 105 N/C. The maximum torque on the dipole is
(A) 0.2 103 N-m
(B) 2.0 103 N-m
(C) 4.0 103 N-m
(D) 1.0 103 N-m
GyaanSankalp

Electric charges and fields


Q.8 The force of attraction between two coaxial electric dipoles whose centres are r m apart varies with distance as
(A) r1
(B) r2
(C) r3
(D) r4
Q.9 An electric dipole is kept on the axis of a uniformly charged ring at distance R / 2 from the centre of the ring. The direction of
the dipole moment is along the axis. The dipole moment is P, charge of the ring is Q and radius of the ring is R. The force on the
dipole is nearly
(A)

4kPQ
3 3R

(B)

4kPQ
3 3R

(C)

2kPQ

(D) zero

3 3R 3

Q.10 Point P lies on the axis of a dipole. If the dipole is rotated by 90 anticlockwise, the electric field vector E at P will rotate by
(A) 90 clockwise
(B) 180 clockwise
(C) 90 anticlockwise
(D) none
ANSWERS
(1) (C)
(2) (D)
(3) (B)
(4) (B)
(5) (C)
(6) (C)
(7) (B)
(8) (D)
(9) (D)
(10) (A)

USEFUL TIPS

1.

The dipole p is parallel to the z-axis. Then, the E x , E y , E z components are


Ex

2.

4.

Ez

p (3z 2 r 2 )
4 0 r 5

The distance dependence of the electric field due to (i) monopole (ii) dipole, and (iii) quadrupole is as follows :

long range

(ii) E

short range

(iii) E

short range
r
r4
r
The coulomb force between two point charges depends only on the charges, their separation and the medium. It is independent
of other charges present.
(i) E

3.

p 3xz
p 3yz
. 5 , Ey
.
4 0 r
4 0 r 5 ,

1
11
The number of lines of force coming out of a unit positive charge is 1.11 10
0

If a cube is placed in uniform electric field the net flux through it will be zero. This also follows from Gauss theorem.
The electric field (E) due to a line of charge is proportional to 1/r.
The electric field (E) due to a point charge is proportional to 1/r 2.
The electric field (E) due to uniformly charged flat sheet is constant at all points. This means it does not depend on distance.
The electric field is uniform in a region, if (a) the number of lines of force crossing unit area normally, is same at all points and (b)
the lines of force are parallel. The first condition (a) makes the magnitude of the field to be the same, while the second condition
(b) makes the direction of the field to be the same at all points.
10. To find the direction of electric field at a point, imagine a unit positive charge at the point. Find the magnitude of force on it. This
gives the magnitude of field. Find the direction of motion of that charge. This gives the direction of electric field.
5.
6.
7.
8.
9.

MISCELLANEOUS SOLVED EXAMPLES


Example 1 :
A point charge +Q is placed at the centroid of an equilateral triangle. When a second charge +Q is placed at a vertex of the triangle,
the magnitude of the electrostatic force on the central charge is 4N. What is the magnitude of the net force on the central charge
when a third charge when a third charge +Q is placed at another vertex of the triangle.
Sol. F k

Q2
4N
r2

Q
r

F=4N

In II case : Charge at centre experiences two forces each


of magnitude 4N at 120.

F 4 4 2 4 4cos120 , F = 4N
2

Q
120

r F=4N

GyaanSankalp

29

Electric charges and fields


Example 2:
Two electrons are a certain distance apart from one another. What is the order of magnitude of the ratio of the electric force
between them to the gravitational force between them.
Sol. Fe

Fg

kq1q 2
r2

Gm1m 2
r

9 109 (1.6 10 19 ) 2
r2

9 1.6 1.6 10 29

6.7 10 11 (9.1 10 31 ) 2
r

r2

6.7 9.1 9.1 10 73


r2

Fe
10 42
Fg

Example 3 :
A particle of mass m and charge q is lying at the mid point of two stationary particles distant 2 and each carrying a charge q. If
the free charged particle is displaced from its equilibrium position through distance x(x <<), then the particle will
(A) move in the direction of displacement
(B) stop at its equilibrium position
(C) oscillate about its equilibrium position
(D) execute S.H.M. about its equilibrium position
Sol. (D). When the charge q is displaced through small displacement x, then the resultant force acting on it is
q
q
q x
Q
P
O
kq 2

F=

( x)2

kq 2

( x) 2

4kq 2
3

x = ma

or

4kq 2
x = 2x
a=
m3

or

a x (Towards O)

The motion of the particle will be S.H.M.


Example 4 :
Millikans drop experiment attempts to measure the charge on a single electron, e, by measuring the charge of tiny oil drops
suspended in an electrostatic field. It is assumed that the charge on the oil drop is due to just a small number of excess electrons.
The charges 3.90 1019 C, 6.50 1019 C and 9.10 1019 C are measured on three drops of oil. Find the charge of an electron.
Sol.
3.9 1019 = n1e, 6.5 1019 = n2e, 9.1 1019 = n3e
n1, n2 and n3 are integers for e = 1.3 1019 .
Example 5 :
Four charges each of magnitude q, are lying at the four corners of a square of side a. How much charge should be placed at the
centre of the square so that the whole system remains in equilibrium?
FA
FB
Sol. Charge Q always remains in equilibrium
F BD
q
D
Condition of equilibrium of charge C
q
C FD
2
2
KQq
Kq
Kq
FO = FBD + FA or
= 2
2 +
a2 / 2
a
2a 2
O

Q
1
or Q = (2 2 1) q
4
q
q
Example 6 :
A
B
The radius of a soap bubble is r and its surface tension is T. If the surface charge density on the bubble is and the excess of
pressure inside it is p, then the value of will be
Sol. In the state of equilibrium
pex + pel = pST

or

p+

2
4T
=
2 0
r

Example 7 :
An electric field line emerges from a positive point charge +q1 at an
angle to the straight line connecting it to a negative point charge q2
(figure). At what angle will the field line enter the charge q2 ?

30

GyaanSankalp

1/2

or = 2 0 4T p
r

Electric charges and fields


Sol. In the immediately proximity of each of point charges, the contribution from the other charge to the total field strength is negligibly
small, and hence the electric field lines emerge from (enter) the charge in a spatially homogeneous bundle, their total number being
proportional to the magnitude of the charge. Only a fraction of these lines gets into a cone with an angle 2 at the vertex near the
charge +q1. The ratio of the number of these lines to the total number of the lines emerging from the charge +q1 is equal to the ratio
of the areas of the corresponding spherical segments.
2 RR (1 cos )

1
(1 cos )
2
4 R
Since electric field lines connect the two charges of equal magnitude, the number of lines emerging from the charge +q1 within the
angle 2 is equal to the number of lines entering the charge q2 at an angle 2. Consequently,
2

| q1 | (1 cos ) | q 2 | (1 cos )
sin

whence

| q1 |
sin
2
2 | q2 |

If | q1 | / | q 2 | sin( / 2) 1 , an electric field line will not enter the charge q2.
Example 8 :
The strength of the electric field produced by charges uniformly distributed
over the surface of a hemisphere at its centre O is E0. A part of the surface is
isolated from this hemisphere by two planes passing through the same diameter
and forming an angle with each other.

Determine the electric field strength E produced at the same point O by the
charges located on the isolated surface (on the mericarp).
Sol. It can easily be seen from symmetry considerations that the vector of the electric
field strength produced by the lobule with an angle lies in the planes of longitudinal and transverse symmetry of the lobule.
Let the magnitude of this vector be E. Let us use the superposition principle and complement the lobule to a hemisphere charged
with the same charge density. For this purpose, we append to the initial lobule another lobule with an angle . Let the
magnitude of the electric field strength vector produced by this additional lobule at the centre of the sphere be E. It can easily be

seen that vectors E and E are mutually perpendicular, and their vector sum is equal to the electric field vector of the hemisphere

at its centre. By hypothesis, this sum is equal to E0. Since the angle between vectors E and E0 is / 2 / 2 ,
we obtain

E E 0 sin

Example 9 :
Two very large thin conducting plates having same cross-sectional area are
placed as shown in figure they are carrying charges Q and 3Q respectively.
The variation of electric field as a function of x (for x = 0 to x = 3d) will be best
represented by

(B)
d

2d

3d

2d

3d

(3d,0)

(C)
d

3Q

(d,0) (2d,0)

(A)

2d

3d

x (D)

2d

3d

Sol. (C).Using the formula for electric field produced by large sheet E
4Q

We get E A 2A (i) ;
0

EB

Q
2A 0

2Q
4Q

(i)
; E C 2A ( i)
2A 0
0
GyaanSankalp

31

Electric charges and fields


Example 10 :
The minimum strength of a uniform electric field which can tear a conducting uncharged thin-walled sphere into two parts is
known to be E0. Find minimum electric field strength capable of tearing the conducting shell of twice as large radius.
Sol. The density of charges induced on the sphere is proportional to the electric field strength : E . The force acting on the
hemispheres is proportional to the field strength :
F SE R 2 E 2
Where S is the area of the hemisphere, and R is its radius. As the radius of the sphere changes by a factor of n, and the field
strength by a factor of k, the force will change by a factor of kn. Since the thickness of the sphere walls remains unchanged, the
force tearing the sphere per unit radius must remain unchanged,

i.e.

1
1
k 2n2

1 and k
.
n
2
n

Consequently, the minimum electric field strength capable of tearing the conducting shell of twice as large radius is E1

E0
2

Example 11 :
The thickness of a flat sheet of a metal foil is d, and its area is S. A charge q is located at a distance from the centre of the sheet
such that d S .
Determine the force F with which the sheet is attracted to the charge q, assuming that the straight line connecting the charge to
the centre of the sheet is perpendicular to the surface of the sheet.
Sol. Since the sheet is metallic, the charges must be redistributed over its surface so that the field in the bulk of the sheet is zero. In the
first approximation, wecan assume that thisdistribution is uniform and has density and over the upper and the lower surface
respectively of the sheet. According to the superposition principle, we obtain the condition for the absence of the field in the bulk
q

of the sheet : 4 2 0
0
0

Let us now take into consideration the nonuniformity of the field produced by the point charge since it is the single cause of the
Sq

force F of interaction. The upper surface of the sheet must be attracted with a force

4 0 2

, while the lower surface must be

Sq

repelled with a force

4 0 ( d) 2

Consequently, the force of attraction of the sheet to the charge is


F

Sq
1
q 2Sd
1

4 0 2 (1 d / ) 2 8 2 05

Example 12 :
In the figure shown, initially the spring of the negligible mass is in underformed state and the block has zero velocity E is a uniform
electric field. Then
(i) The maximum speed of the block will be

QE
mK

(ii) The maximum speed of the block will be

2QE
mK

2QE
QE
(iv) The maximum compression of the spring will be
K
K
E
(A) only (i) and (ii) are correct (B) only (i) and (iv) are correct
(C) only (ii) and (iii) are correct
Q
K
(D) only (ii) and (iv) are correct
Sol. (B). Speed will be maximum when acceleration becomes zero
smooth m

(iii) The maximum compression of the spring will be

i.e. , when KX = EQ X

EQ
K

1
1
2
2
By work-energy theorem : Wall KE EQX KX mv
2
2

32

GyaanSankalp

Electric charges and fields


QE
EQ
: Vmax
mK
K
compression will be maximum when velocity becomes zero.
Substituting

1
2EQ
Wall KE EQX KX 2 0 ; X max
2
K

Example 13 :
Two balls of charge q1 and q2 initially have a velocity of the same magnitude and direction. After a uniform electric field has been
applied during a certain time, the direction of the velocity of the first ball changes by 60, and the velocity magnitude is reduced
by half. The direction of the velocity of the second ball changes thereby by 90. In what proportion will the velocity of the second
ball change ? Determine the magnitude of the charge-to-mass ratio for the second ball if it is equal to k1 for the first ball. The
electrostatic interaction between the balls should be neglected.
Sol. Let v1 and v2 be the velocities of the first and second balls after the removal of the uniform electric field. By hypothesis, the angle
between the velocity v is 60. Therefore, the change in the momentum of the first ball is p1 q1Et m1vsin 60
Here we use the condition that v1 = v/2, which implies that the change in the momentum p1 of the first ball occurs in a direction
perpendicular to the direction of its velocity v1.
Since E || p1 and the direction of variation of the second ball momentum is parallel to the direction of p1 , we obtain for the
velocity of the second ball. (it can easily be seen that the charges on the balls have the same sign.)

v 2 v tan 30

3
The corresponding change in the momentum of the second balls is
p 2 q 2 Et

m2v
cos30

q1
m1 sin 60
Hence we obtain, q m / cos 30 ;
2
2

q2
4 q1 4

k1
m 2 3 m1 3

Example 14 :
An infinite long plate has surface charge density . As shown in the figure a
point charge q is moved from A to B. Net work done by electric field is
q
(A) 2 (x1 x 2 )
0

q
(B) 2 (x 2 x1 )
0

q
(C) (x 2 x1 )
0

q
(D) (2r r)
0

Sol. (A). Wnet qE.d

B(x1,0)

A(x 2,0)


where E
i , d = (x1 x2)
2 0

Example 15 :
Estimate the upper limit of the error made in calculating the force of interaction between charged spherical conductors with the aid
of the Coulomb law. The radii of the spheres are r0, the distance between their centres is r.
Sol. It is evident from figure for the case of charges of different
signs that the actual of interaction of the charges is greater
than it would have been, if the charges were concentrated
in the centres of the spheres, and less than if the charges
were concentrated at the nearest points of the spheres :
q2
4 0 r

q2
4 0 (r 2r0 )2

The absolute error is less than the difference between the boundling values, the relative error being less than the ratio of this
difference to the minimum force. Hence,
GyaanSankalp
33

Electric charges and fields

F2 F1 r 2 (r 2r0 ) 2 4r0 (r r0 )

F1
(r 2r0 ) 2
(r 2r0 ) 2

Example 16 :
A dipole is placed in the field of a point charge, the distance between the dipole and the field source being much greater than the
dipole separation. Find the force acting on the dipole and the torque, if the dipole is arranged :
(a) perpendicular to the line of force, (b) in the direction of the lines of force.
Sol. (a) It is evident from figure (a) that in this case a force couple acts on the dipole.
The resultant of the force is zero. To obtain the torque, multiply the force by
the dipole separation.
(b) It is evident from figure (b) that in this case to forces act on the dipole in the
direction of the radius vector. Hence it is clear that the torque is zero, and the
resultant force acts in the direction of the radius to the source.
The resultant can be found by two methods.
One may use the Coulomb law :
Qq

F F F

4 0 (r / 2)

2Qrq

=
4 0 (r / 2)
4 0 (r 2 2 / 4) 2
Qq

Nothing that the problem stipulates that << r, we obtain


F

2Qrq
4 0 r

2Qp e
4 0 r3

The same result may be obtained from formula, if the derivative is substituted for the ratio of increments. We have
F pe

2Qpe
dE
d Q
pe

.
2
dr
dr 4 0 r
4 0 r3

Alternatively, use electric field expression due to dipole.


Example 17 :
A cube of edge a metres carries a point charge q at each corner. Calculate the resultant force on any one of the charges.
Sol. Let us take one corner of cube as origin O(0, 0, 0) and the opposite corner as P(a, a, a). We will calculate the electric field at P due
to the other seven charges at corners.
Expressing the field of a point charge in vector form

q
E
r
C
4 0 r 3
Z
P

E1

q
q
AP BP CP
aj ak ai
4 0 a 3
4 0 a 3

(i) Field at P due to A, B, C

(ii) Field at P due to D, E, F

F
D

Now that DP = EP = FP = a 2
q

E2

4 0 a 2

DP EP FP

aj ak ai aj ai ak
2a )

4 0 ( 2

q
4 0 2a

(iii)Field at P due to O : OP a 3

E3

4 0 a 3

OP

; E3

q
2

4 0 (3 3a )

(ai aj ak ) ; E 3

q
4 0 (3 3a 2 )

Resultant Field at P

q (i j k )
1
1
E E1 E 2 E 3 ; E

2
4 0 a

34

GyaanSankalp

3 3

outward along OP

i j k

i j k

Electric charges and fields


Force on charge at P is F = q E F

q2 3
4 0a 2

1
1

2 3 3

Outwards along diagonal OP


Example 18 :
An electron is projected along the axis midway between the plates of a cathode ray tube with an initial velocity of
2 107 ms1. The uniform electric field between the plates has an intensity of 20,000 NC1 and is upward.
(a) How war below the axis has the electron moved when it reaches the end of the plates ?
(b) At what angle axis is it moving as it leaves the plates ?
Sol. The electric field is upward, so the electric force on the electron is downward. Let the x-axis lie along the initial velocity. The xcomponent of the electrons velocity remains constant, and the time for the electron to traverse the plates is
t

(x x 0 )
0.04m

2.0 10 19 s
v0x
2 10 7 ms1

In the downward (y) direction the acceleration of the electron is produced by the electric force.

ay

Fy
me

E | q | (20, 000NC1 ) (1.60 1019 C)

= 3.51 1015 ms 2
me
9.11 1031 kg

The downward displacement of the electron as it passes between the


plates is then given by
1
a yt2
2

y y 0 v 0y t
y y0
tan

tan

1
(3.51 1015 ms2 ) (2.0 109 s) 2 = 7.02 103 m = 0.702 m
2

vy

6
1
v x 2 107 ms 1 , v y voy a y t 7.02 10 ms

vx
vy

0.351 19.3

vx

Example 19 :
Two small balls with equal but opposite charges are secured in a horizontal plane at a distance a from each other. A third charged
ball is suspended on a string. The point of suspension is so move that the third ball, when in a state of equilibrium, is precisely
above the first ball at a distance a from it, and then it is so moved that the third ball is above the second one. Find the angles
through the string is deflected the balls if in second case it is double the first case.
Sol. The conditions of equilibrium of the suspended ball give the following equations for the two cases being considered :
T1 sin 1

KQQs

T1 cos 1
T2 cos 2

2a

2
0
2

KQQs 2
2 KQQs
0

mg 0 ; T2 sin 2
2
2
2a 2 2
a

KQQs
2a

KQQs
a2

KQQs
2a 2

2
mg 0
2

where T1 and T2 are the tensions of the thread, 1 and 2 the angles of
deflection of the thread, +Q and Q the charges of the fixed balls, +QS
the charge of the suspended ball, and mg is the weight of the suspended
ball.
Upon excluding the unknowns from the above simultaneous equations, we get

cot 1 cot 2 cot 1 cot 21 2 ( 2 1)


GyaanSankalp

35

Electric charges and fields


whence,

cot 1 2(2 2 1) 35 16 2

Thus,

1 756' and 2 1552 ' (From Trigonometric Table)

when

mg

KQQs
2
1 4
2
a

when

mg

KQQs
2
1 4
2
a

1 8204' and 2 16408'

and

Example 20 :
The negative charge q2 is fixed while positive charge q1 as well
as the conducting sphere S is free to move. If the system is
Conducting
q2
released from rest
shell 'S'
(A) both S and q1 move towards left
q1
(B) q1 moves towards right while S moves towards left
/////////////
(C) q1 remains at rest, S moves towards left
(D) both q1 and S remain at rest
Sol. (C).
Net force on q1 is zero, while that on the conducting sphere is towards the left due to the attraction of q2.
After sometime : q1 will not be at the centre and due to this the charge on the inner surface will not remain uniformly charged.
q1 will experience net force towards right
more negative charge will be present on that portion of the shell which is near to q1.
Example 21 :
Find the electric field inside a sphere which carries a charge density proportional to the distance from the origin = r
(is a constant).
Sol. We can consider all the charge inside the sphere to be concentrated
on the center of sphere.
dx
Consider an elementary shell of radius x and thickness dx.
E

K dq
r2

K 4x 2dx (x)
r2

r 2

4 0

Example 22 :
Figure shows two large cylindrical shells having uniform linear charge
densities + and . Radius of inner cylindrical is a and that of outer
cylinder is b. A charged particle of mass m, charge q revolves in a
circle of radius r. Find its speed v. (Neglect gravity and assume the radii
of both the cylinders to be very small in comparison to their length)

r
2k
Sol. Electric field between the two cylinders =
r
2kq
Force on charge q
r

This force is centripetal force


1 q
v 2 4 m
0

36

GyaanSankalp

v
a
b

2kq mv2

r
r

q
2 0 m

Electric charges and fields


Example 23 :
Three identical positive charges Q are arranged at the vertices of an equilateral triangle. The side of the triangle is a. Find the
intensity of the field at the vertex of a regular tetrahedron of which the triangle is the base.
Sol. Each charge creates at point D a field intensity of E1

KQ

. The total intensity


a2
will be the sum of three vectors (fig.). The sum of the horizontal components of
these vectors will be zero, since they are equal in magnitude and form angles of
120 with each other. The vectors from angles of 90 with the vertical,
where is the angle between the edge of the tetrahedron and the altitude h of
triangle ABC.
KQ

The vertical components are identical each being equal to


It follows from triangle ADE that sin

a2

sin .

2
.
3

Therefore, the sought intensity of the field is

E 6

KQ
a2

Example 24 :
A molecule is at a distance r from the axis of a charged infinitely long metallic cylinder. Find the force acting on the molecule if the
2q
(q is the charge per unit of cylinder length) and the molecule has
r
the form of a dumb-bell with a length and with charges +Q and Q at its ends.
Sol. The molecule will be attracted to the charged cylinder. The force of attraction is

intensity of the cylinder field is expressed by the formula E

1
2qQ
1
F 2qQ

r r r (r )

In this expression we want neglect the quantity ( 108 cm) as compared with r (r cannot be smaller than the cylinder radius).
We finally obtain

2qQ
r2

Example 25 :
Two molecules of equal mass are at a certain distance from the axis of a charged cylinder. One molecule has a constant electric
moment p = Q . An elastic force acts between the charges of the other molecule, i.e., the distance is determined from the
expression QE = k , where E is the mean intensity of the field acting on the molecule and k is a proportionality factor. First the
electric moments of the molecules are the same and their velocities are zero.
Which molecule will reach the surface of the cylinder quicker under action of the force of attraction.
Sol. At the initial moment the forces acting on both molecules are identical. When the molecules approach the cylinder, the force F1
acting on the molecule with a constant electric moment grows in proportional to 1/r2.
F1

2qQ
r2

The force F2 that acts on the elastic molecule grows faster, in proportion to 1/r, owing to the continuous increase in the electric
moment of this molecule (F2 4q 2 Q 2 / kr 3 )
The masses of the molecules are the same, and for this reason the acceleration of the second molecule when it approaches the
cylinder grows faster than that of the first one and it will reach the surface of the cylinder quicker.
Example 26 :
A charged particle of mass m and having a charge Q is placed in an electric field E which varies with time as E = E0 sin t. What
is the amplitude of the S.H.M. executed by the particle.
Sol. For a particle undergoing SHM with an amplitude A and angular frequency , the maximum acceleration = 2A
Here the maximum force on the particle = QE0
maximum acceleration =

QE 0
2A
m

QE 0
m 2
GyaanSankalp

37

Electric charges and fields

QUESTION BANK
EXERCISE - 1
CONCEPTUAL QUESTIONS
Q.1
Q.2
Q.3
Q.4
Q.5
Q.6

Q.7

Q.8
Q.9

Under what conditions can a brass rod be charged ?


An electric charge on a conducting sphere has to be
divided into three equal parts. How can this be done ?
Why is a metal chain that reaches the ground fixed to a
lorry for transporting petrol ?
Can a positive charged be obtained on the electroscope
using a negatively charged ebonite rod ?
What will happen to the surface charge density on a metal
sheet rolled into a cylinder ?
An elder ball is tied to a silk thread. What will happen
when an electrostatically charged rod is brought close to
it ?
Do the electric field vector and the vector of the force
exerted on a charge by an electric field always have the
same directions ?
Why is metal cap sometimes put on a vacuum tube ?
Can electric charges be separated on
(a) conductor, (b) a dielectric ?

Q.10 A thin insulator rod is placed between two unlike point


charges +q1 and q2 (fig.). How will the force acting on the
charges change ?

Q.11 A sphere carries a uniformly distributed electric charge.


Prove that the field inside the sphere is zero.
Q.12 If only one charged body is available, can it be used to
obtain a charge exceeding many times in absolute magnitude that which it itself has ?
Q.13 Can two likely-charged balls be attracted to each other ?
Q.14 A small metallic sphere is brought in contact in turn with
points A, B, C of a charged body. Find the approximate
charge on the sphere after each contact as would be indicated by bringing the sphere in contact with an electroscope. Do the leaves of the electroscope diverge at the
same angle in all three cases?

EXERCISE - 2
tex of the triangle, the magnitude of the electrostatic force
on the central charge is 8N. The magnitude of the net force
on the central charge when a third charge +Q is placed at
another vertex of the triangle is
(A) zero
(B) 4N

ONLY ONE OPTION IS CORRECT


Q.1

A semi-infinite insulating rod has linear charge density .


The electric field at the point P shown in figure is

++++++++++++++
A
B

Q.5

2 2

(A)

Q.2

Q.3

Q.4
38

(4 0 r)2

2 2

at 45 with AB (B)

4 0 r 2

(C)

2
at 45 with AB
4 0 r

(D)

2
at perpendicular to AB
4 0 r

at 45 with AB

A certain charge Q is divided into two parts q and (Q-q).


For the maximum coulomb force between them, the ratio
(q/Q) is :
(A) 1/16
(B) 1/8
(C) 1/4
(D) 1/2
The force between two point charges + Q and Q placed
r distance apart is f1 and force between two spherical
conductors, each of radius R placed with their centres r
distance apart charged with charge +Q and Q is f2. If the
separation r is not much larger than R, then :
(A) f1 > f2
(B) f1 = f2
(C) f1 < f2
(D) f2 = (r/R) f1
A point charge +Q is placed at the centroid of an equilateral triangle. When a second charge +Q is placed at a verGyaanSankalp

(C) 4 2N
(D) 8N
Two identical metallic blocks resting on a frictionless horizontal surface are connected by a light metallic spring having the spring constant 100 N/m and an unstretched length
of 0.2m, as shown in figure 1. A total charge Q is slowly
placed on the system, causing the spring to stretch to an
equilibrium length of 0.3m, as shown in figure 2. The value
of charge Q, assuming that all the charge resides on the
blocks and that the blocks are like point charges, is

Figure 1

Figure 2

Q.6

(A) 10 C
(B) 15 C
(C) 20 C
(D) 30 C
A particle of charge q and mass m moves in a circle of
radius r around an infinitely long line charge on linear
charge density +. Then time period will be
[where k

1
]
4 0

Electric charges and fields


(A) T 2r
2
(B) T

Q.7

Q.8

Qa

m
2kq

(A)

4 2 m 3
r
2kq

(C) T

1
2r

(D) T

1
m
2r 2kq

y
A large uniformly charged
(negative) plate is placed
+Q
in xz plane and a positive

point charge is fixed on


+
the y-axis. A dipole is positioned in between with
x
its axis along y-axis, as
plate
shown. The dipole initially
moves in
(A) negative y-direction
(B) negative x-direction
(C) positive x-direction
(D) positive y-direction
A ring of radius R is made out of a thin metallic wire of area
of cross section A. The ring has a uniform charge Q
distributed on it. A charge q0 is placed at the centre of the
ring. If Y is the youngs modulus for the material of the ring
and R is the change in the radius of the ring, then :

(C) R

q 0Q
2 2
8 0 RAY

q 0Q
(B) R 4 RAY
0

(D) R

(C)

Q.12 When three electric dipoles are near each other, they each
experience the electric field of the other two, and the three
dipole system has a certain potential energy. Figure below
shows three arrangements (A), (B) and (C) in which three
electric dipoles are side by side. All three dipoles have the
same electric dipole moment magnitude and the spacings
between adjacent dipoles are identical. If U1, U2 and U3
are potential energies of the arrangements (A) , (B) and (C)
respectively, then

(a)

8 0 RAY

4Rq
i
(B)

q + + + q

X
R

2Rq
2Rq
i (D)
i

Q.11 Six charges are placed at the


+Q
vertices of a regular hexagon
as shown in the figure.
The electric field on the line +Q
passing through point O and
perpendicular to the plane of
+Q
the figure at a distance of
x ( >>a) from O is

2g

(D)
Q

=45

mg
q

m
3g
5g
m
(D)
+q
l
l
Q.15 A dipole of dipole moment p is kept at the centre of radius
R and charge Q. The dipole moment has direction along
the axis of the ring. The resultant force on the ring due to
the dipole is

(B)

(C)

(C)

(A) zero

(C)

(c)

of 100 units in x-y plane is :


(A) 800 units
(B) 300 units
(C) 400 units
(D) 1500 units
Q.14 A horizontal electric field (E = (mg)/q) exists in space as
shown in figure and a mass m is attached at the end of a
light rod. If mass m is released from the position shown in
figure find the angular velocity of the rod when it passes
through the bottom most position :
(A)

A charge is situated at a certain distance from an electric


dipole in the end-on position experiences a force F. If the
distance of the charge is doubled, the force acting on the
charge will be :
(A) F/4
(B) F/8
(C) 2F
(D) F/2
Y
Q.10 The dipole moment of the
given charge distribution is :

(b)

(A) U1 > U3 > U2


(B) U1 > U2 > U3
(C) U1 > U2 = U3
(D) U1 = U2 = U3
Q.13 In a region of space the electric field is given by

E 8i 4j 3k . The electric flux through a surface of area

q 0Q
2

(D) zero

0 x 3

Q.9

4Rq
i
(A)

0 x 3

3Qa
q

2kq
m

q 0Q
(A) R 4 RAY
0

0 x

2Qa

(B)

(B)

kPQ
R3

2kPQ
R3
kPQ
R3

only if the charge is uniformly distributed on the

ring
Q.16 Two point charges +q and q are held fixed at (-d, 0) and (d,
0) respectively of a (X, Y) coordinate system. Then

(A) The electric field E at all points on the X-axis has the
GyaanSankalp

39

Electric charges and fields


same direction.

(B) E at all points on the Y-axis is along i


(C) Work has to be done in bringing a test charge from
infinity to the origin.
(D) The dipole moment is 2qd directed along i
Q.17 A particle of charge q and mass m moves rectilinearly under the action of electric field E = A Bx, where A and B are
positive constants and x is distance from the point where
particle was initially at rest then the distance traveled by
the particle before coming to rest and acceleration of particle at that moment are respectively :
2A
,0
(A)
B

Q.19

Q.20

Q.21

qA
m

2A qA
2A qA
,
,
(D)
B
m
B
m
A charge q is placed at O in
the cavity in a spherical
uncharged conductor. Point S
is outside the conductor.
If the charge is displaced from
O towards S, still remaining
within the cavity,
(A) electric field at S will increase
(B) electric field at S will decrease
(C) electric field at S will first increase and then decrease
(D) electric field at S will not change
Which of the following statements is true
(A) The electric field due a point charge can be same at two
points.
(B) The electric field increases continuously as one goes
away from centre of a solid uniformly charged sphere
(C) The electric field of force of the electric field produced
by the static charges from closed loops
(D) The magnetic lines of force of magnetic field produced
by current carrying wire from closed loops
An electron of mass me initially at rest moves through a
certain distance in a uniform electric field in time t1. A proton of mass mp also initially at rest takes time t2 to move
through an equal distance in this uniform electric field.
Neglecting the effect of gravity the ratio of t2/t1 is nearly
equal to :
(A) 1
(B) (mp/me)1/2
1/2
(C) (me/mp)
(D) 1836
The variation of electric field between two point charges
along the line joining the charges is given in figure. Then
which is/are correct ?

(C)
Q.18

(B) 0,

(A) Q1 is +ve and Q2 is ve (B) Q1 is +ve and Q2 is +ve


(C) | Q1 | | Q2 |
(D) | Q1 | | Q2 |
Q.22 A largest sheet carries
+
uniform surface charge

density A rod of length 2


O )
has a linear charge density
on one half and on the

other half.
+
The rod is hinged at mid point
O and makes angle with the normal to the sheet. The
torque experienced by the rod is
(A)

2
cos
2 0

2
(B) cos
0

2 sin
(C)
2 0

sin 2
(D)
0

(A)

(C)

(B)

q
(D) 4 R
0

2 0 R
2

4 0 R

////////////////

0 R

////////////////

////////////////

////////////////

////////////////
B

(A)
(B)
(C)
(D)
GyaanSankalp

Q.24 Five styrofoam balls are suspended from insulating threads.


Several experiments are performed on the balls and the
following observations are made
(i) Ball A repels C and attracts B
(ii) Ball D attracts B and has no effect on E
(iii) A negatively charged rod attracts both A and E.
An electrically neutral styrofoam ball gets attracted if
placed nearby a charged body due to induced charge.
What are the charges, if any, on each ball ?

40

+
+
+
+ +
+ ++
+ +
+
+ R ++
+
+
+ ++
+
+
+
+
+
+

Q.23 Find the force experienced by the


semicircular wire charged with a
charge q, placed as shown in figure.
Radius of the wire is R and the line
of charge with linear charge density
is passing through its centre and
perpendicular to the plane of wire.

A
+
+
+

C
+
+
+

D
0
+
0
0

E
+
0
0
0

Electric charges and fields


Q.25 The electric force on 2C
charge placed at the centre O
of two equilateral triangles
each of side 10cm, as shown
in figure is F.
If charges A, B, C are each 2C
and charges D, E and F are
each 2C, then F is :
(A) 2 1.6N
(B) 64.8N
(C) 0
(D) 43.2N
Q.26 The electric field in a region of space is given by

E 5i 2j N / C . The electric flux due to this field through


an area 2m2 lying in the YZ plane, in S.I. unit, is
(A) 10
(B) 20
(C) 10 2

(D) 2 29

p
k are located at (0, 0, 0) and
2
(1m, 0, 2m) respectively. The resultant electric field due to
the two dipoles at the point (1m, 0, 0) is

Q.27 Two point dipoles pk and

9p
(A) 32 k
0

7p
(B) 32 k
0

7p
(C) 32 k
0

(D) None of these

Q.28 Four charges are rigidly


fixed along the Y-axis as
shown. A positive charge
approaches the system
along the X-axis with initial speed just enough to
cross the origin. Then its
total energy at the origin
is

Q.31 Two point point charges exert on each other a force F when
they are placed r distance apart in air. When they are placed
R distance apart in a medium of dielectric constant K, they
exert the same force. The distance R equals
(A)

(B)

r
K

(C) rk
(D) r K
Q.32 A ring of radius R is placed in the plane with its centre at
origin and its axis along the x-axis and having uniformly
distributed positive charge. A ring of radius r (<< R) and
coaxial with the ring is moving along the axis with constant
velocity then the variation of electric flux () passing
through the smaller ring with position will be best represented by

x
R

y
2 2q
q

(A)
v Q

1 q
2

(B)

2q

r
(D) 0
4

Q.30 Find the electric flux crossing the wire frame ABCD
of length , width b and
d
whose centre is at a disO
tance OP = d from an infinite line of charge with linear charge density .
Consider that the plane
of frame is perpendicular to the line OP.

1 b
(A) tan 2d
0

1 b
(D) 2 tan 4d
0

(A) zero
(B) positive
(C) negative
(D) data insufficient
Q.29 A circle of radius r has a linear charge density = 0 cos2
along its circumference. Total charge on the circle is
(A) 0(2r)
(B) 0(r)

r
(C) 0
2

1 b
(C) tan 4d
0

(C)

(D)

Q.33 A point charge q is placed at origin. Let EA , EB and EC be


the electric field at three points A (1,2,3), B (1, 1, 1) and
C (2, 2, 2) due to charge q. Then

(A) E A E B
(B) | E B | 4 | EC |

(C) EA || EB
(D) | E B | 2 | EC |
Q.34

A small sphere of mass m and carrying a charge q is


attached to one end of an insulating thread of length a,
the other end of which is fixed at (0,0) as shown in the

figure. There exists a uniform electric field E E0 j in the

1 b
(B) 2 tan 2d
0
GyaanSankalp

41

Electric charges and fields


region.
The minimum velocity which should
be given to the sphere at (a,0) in the
direction shown so that it is able to
complete the circle around the origin
is (There is no gravity) :
(A)

5qE 0 a
m

(A) (1 2 )/ 0

y
v
O

3qE 0 a
m

(B)

qE 0 a
qE 0 a
(C)
(D) 2
m
m
Q.35 A charge Q is placed at the
centre of an imaginary hemiQ
spherical surface. Using symmetry arguments and the
Gausss law.
The flux of the electric field
due to this charge through the surface of the hemisphere
(figure) is
Q
(A) 2
0

Q
(B)
0

2Q
(C)
0

2Q
(D) 3
0

Q.36 A particle of mass m and charge Q is placed in an electric


field E which varies with time t as E = E0sin wt. It will
undergo simple harmonic motion of amplitude
(A)

(C)

QE 20

(B)

m 2

QE 0
2

(D)

QE 0
m 2

QE 0
m

q y
m
Q.37 An equivalent triangle wire
x
frame of side L having 3 point
charges at its vertices is kept
in x-y plane as shown.
Component of electric
field due to the configuration
q
2q
in z direction at (0, 0, L) is [origin is centroid of triangle]

(A)
(C)

9 3 kq
8L2
9 kq

(B) zero

(D) None
8L2
Q.38 A particle of mass m and charge q is placed at rest in a
uniform electric field E and then released. The KE attained
by the particle after moving a distance y is
(A) qEy2
(B) qEy
(C) qE2y
(D) q2Ey
Q.39 If electric flux entering and leaving on enclosed surface is
1 and 2 respectively, the electric charge inside the
enclosed surface will be
42

GyaanSankalp

(B) (2 1 )/ 0

(C) (1 2 )/ 0
(D) ( 2 1 ) / 2 0
Q.40 Two particles A and B having equal charges are placed at a
distance d apart. A third charged particle placed on the
perpendicular bisector at a distance x will experience the
maximum Coulombs force when
(A) x d / 2

(B) x = d/2

(C) x d / 2 2
(D) x d / 3 2
Q.41 A charged particle of mass m and charge q initially at rest is
released in an electric field of magnitude E. Its kinetic energy
after time t will be
(A)

(C)

2E 2 t 2
mq
Eq 2 m
2

2t
Q.42 A square surface of
side L metres is in the
plane of the paper. A
uniform electric field

E (volt/m), also in the

(B)

E 2q 2 t 2
2m

(D)

Eqm
t

plane of the paper, is limited only to lower half of the square


surface, (see figure). The electric flux in SI units associated with the surface is
(A) EL/2
(B) zero
(C) EL
(D) EL/(20)
Q.43 A mass particle (mass = m and charge = q) is placed between two point charge q. If these charge displaced 2L.
distance the frequency of oscillation of mass particle, if it
is displaced for a small distance
1

(A)

q
2

m 0 L3

(C)

q
2

4m 0 L3

(B)

q
2

2m 0 L3

(D)

q
2

16 m0 L3

Q.44 Two equal negative charges -q are placed at point (0, a)


on y-axis, one positive charge q is placed at x = 2a, this
charge will(A) Execute S.H.M. about the origin
(B) Oscillate but not execute S.H.M.
(C) Move towards origin and will become stationary
(D) S.H.M. along x axis
Q.45 An electric charge q is placed
q
at the centre of the open end
of a hollow cylindrical vessel
of length and radius r.
Then the electric flux coming

out of the surface of the cylinder isq


(A)
0

Electric charges and fields


q
2
(B) (r 2r)
0

q
(C) 2
0

(C)

(D) Zero
Q.46 The electrical breakdown of air occurs at E = 3000 volt/mm.
Then the maximum charge that can be given to a sphere of
diameter 5 metre will be nearly- (in coulomb)
(A) 2.1 103
(B) 0.83 103
(C) 4.6 102
(D) 2.1 106
Y
Q.47 The electric field in the region
shown here is given by
E x i volt/m. Then the
total electric flux through the
cube of side a is-

(A) Zero
(C) a5/2
(D) a3/2
Q.48 Three identical spheres each having a charge q and radius
R, are kept in such a way that each touches the other two.
Find the magnitude of the electric force on any sphere due
to other two.
1
3q
(A)
4 0 4 R
1
3q
(C)
2 0 4 R

1
3q
(B)
4 0 2 R

(D) None

Q.49 If two infinite oppositely


charged plates with surface
charge density || on each
plate are kept perpendicular
to each other as shown in
above figure, then a charge q at P will experience a force
(A)

q
towards the
2 0

(B)

q
away from horizontal plate
2 0

2 0
q
2 0

towards the origin O

making an angle 45 with respect to vertical

Q.50 The electric dipole of moment p pi is kept at a point

(x, y) in an electric field E 4xy 2 i 4x 2 y j . The force on


the dipole is :
(A) zero

(B)

pyx
x 2 y2

(B) a2

(D)

(C) 4py y 2 4x 2
(D) can not be calculated
Q.51 Figure shows a metal body of
mass M charged positively. P
is a point in front of the body
. The electric field at point P
due to the body M is EP .
Now a negative charge q
is placed at point P and it
experiences a force F . Then :
(A) EP = F/q
(B) EP < F/q
(C) EP > F/q
(D) None of these
Q.52 There is a non-uniform electric
field along x-axis as shown in
figure. The field increases at
a uniform rate along +ve xaxis. A dipole is kept inside
the field as shown.
Which one of the following
statements is correct for dipole ?
(A) dipole moves along positive x-axis and rotates
clockwise
(B) dipole moves along negative x-axis and rotates
clockwise
(C) dipole moves along positive x-axis and rotates
anticlockwise
(D) dipole moves along negative x-axis and rotates anticlockwise

EXERCISE - 3
ONE OR MORE THAN ONE CHOICE MAY
BE CORRECT
Q.1

A thin walled spherical conducting shell S of radius R is


given charge Q. The same amount of charge is also placed
at its centre C. Which of the following are correct
Q
(A) On the outer surface of S charge density =
2R 2
(B) The electric field is zero at all points inside S

Q.2

(C) At a point just outside S, the electric field is double, the


field at a point just inside S.
(D) At any point inside S, the electric field is inversely
proportional to the square of distance from C.
Electric field, due to an infinite line of charge, as shown in
figure at a point P at a distance r from the line is E. If one
half of the line of charge is removed from either side of
point A, then :

GyaanSankalp

43

Electric charges and fields

Q.3

(A) Electric field at P will have magnitude E/2


(B) Electric field at P in x direction will be E/2.
(C) Electric field at P in y direction will be E/2.
(D) none of these

C . m is
An electric dipole moment p (2.0i 3.0j)

105 NC1
placed in a uniform electric field E (3.0i 2.0k)

(A)The torque that E exerts on p is

(0.6i 0.4j 0.9k)Nm

Q.4

Q.5

Q.6

Q.7

44

(B) The potential energy of the dipole is 0.6 J


(C The potential energy of the dipole is 0.6 J
(D) If the dipole is rotated in the electric field, the maximum
potential energy of the dipole is 1.3 J
Mark the correct options
(A) Gausss law is valid only for uniform charge distributions
(B) Gausss law is valid only for charges placed in vacuum
(C) The electric field calculated by Gausss law is the field
due to all the charges.
(D) The flux of electric field through a closed surface due
to all the charges is equal to the flux fue to the charges
enclosed by the surface.
An electric field converges at the origin whose magnitude
is given by the expression E = 100rNt/Coul., where r is the
distance measured from the origin.
(A) total charge contained in any spherical volume with its
centre at origin is negative.
(B) total charge contained at any spherical volume, irrespective of the location of its centre, is negative.
(C) total charge contained in a spherical volume of radius 3cm
with its centre at the origin has magnitude 31013 C.
(D) total charge contained in a spherical volume of radius
3 cm with its centre at the origin has magnitude 3 109 C.
Select the correct alternative
(A) The charge gained by the uncharged body from a
charged body due to conduction is equal to half of the
total charge initially present.
(B) The magnitude of charge increases with the increase in
velocity of charge.
(C) Charge cannot exist without matter although matter
can exist without charge
(D) Between two non-magnetic substances repulsion is
the true test of electrification (electrification means body
has net charge)
The electric field intensity at a point in space is equal in
magnitude to
(A) Magnitude of the potential gradient there
GyaanSankalp

(B) The electric charge there


(C) The magnitude of the electric force, a unit charge would
experience there
(D) The force, an electron would experience there
Q.8 Figure shows a charge q
D
placed at the centre of a hemi- B
sphere. A second charge Q is
placed at one of the positions
A, B, C and D.
In which position(s) of this C
A
q
second charge, the flux of the electric field through the
hemisphere remains unchanged ?
(A) A
(B) B
(C) C
(D) D
Q.9 An electric dipole is placed at the centre of a sphere, mark
the correct options
(A) The flux of the electric field through the surface is zero
(B) The electric field is zero at every point of the sphere
(C) The electric field is not zero anywhere on the sphere
(D) The electric field is zero an a circle on the sphere.
Q.10 A large nonconducting sheet M is given a uniform charge
density. Two uncharged small metal rods A and B are
placed near the sheet as shown in figure.
A

(A) M attracts A
(C) A attracts B

(B) M attracts B
(D) B attracts A
Q.11 A uniform electric field of strength Ej exists in a region.
An electron (charge e, mass m) enters a point A with
velocity Vj . It moves through the electric field and exits at
point B. Then
V

(0, 0)

(A) E

A(a, 0)

B(2a, d)

2amv2
j
ed 2

(B) Rate of work done by the electric field at B is

4ma 2 v3
d3

(C) Rate of work by the electric field at A is zero


2av
i vj
d
Q.12 An oil drop has a charge 9.6 1019 C and has a mass
1.6 1015 gm. When allowed to fall, due to air resistance

(D) Velocity at B is

Electric charges and fields


force it attains a constant velocity. Then if a uniform electric field is to be applied vertically to make the oil ascend
up with the same constant speed, which of the following
are correct. (g = 10 m/s) (Assume that the magnitude of
resistance force is same in both the cases)
(A) the electric field is directed upward
(B) the electric field is directed downward
(C) the intensity of electric field is

1
102 NC1
3

1
105 NC1
6
Q.13 An electric dipole is kept in the electric field produced by
a point charge

(D) the intensity of electric field is

(A) dipole will experience a force


(B) dipole can experience a torque
(C) dipole can be in stable equilibrium
(D) it is possible to find a path (not closed) in the field on
which work required to move the dipole is zero.
y
Q.14 Point charges are located on
the corner of a square as
1C
+1C
shown.
Find the components of electric
field at any point on the z-axis
x
which is axis of symmetry of
the square
1C
(A) Ez = 0
(B) Ex = 0
+1C
(C) Ey = 0
(D) None of these

EXERCISE - 4
ASSERTION AND REASON QUESTIONS

Q.1

Q.2
Q.3

Q.4

Q.5

Q.6

Note : Each question contains STATEMENT-1 (Assertion)


and STATEMENT-2 (Reason). Each question has 5 choices
(A), (B), (C), (D) and (E) out of which ONLY ONE is correct.
(A) Statement-1 is True, Statement-2 is True; Statement-2
is a correct explanation for Statement-1.
(B) Statement-1 is True, Statement-2 is True; Statement-2
is NOT a correct explanation for Statement-1.
(C) Statement -1 is True, Statement-2 is False.
(D) Statement -1 is False, Statement-2 is True.
(E) Statement -1 is False, Statement-2 is False.
Statement 1 : A positive point charge is brought in an
electric field, the electric field at nearby point will increase.
Statement 2 : Electric field produce by charge may favour
the existing electric field.
Statement 1 : Flux through a closed surface is zero.
Statement 2 : Total charge inside the surface must be zero.
Statement 1 : Consider two
identical charges placed distance 2d apart, along x-axis.
The equilibrium of a positive
test charge placed at the point O midway between them is
stable for displacements along the x-axis.
Statement 2: Force on test charge is zero.
Statement 1 : A parallel beam of electrons is shot into a
uniform strong electric field initially parallel to and then
against the field with a small initial speed. Then the beam
tends to spread out at the beginning and narrows down later.
Statement 2: The total energy of the beam is conserved.
Statement 1 : An ellipsoidal cavity is carved within a perfect conductor. A positive charge q is placed at the centre
of the cavity. The points A and B are on the cavity surface.
Potential at A = potential at B.
Statement 2 : Surface of charge conductor is always equipotential.
Statement 1 : A deuteron and an -particle are placed in an
electric field. If F1 and F2 be the forces acting on them and
a1 and a2 be their accelerations respectively then, a1 = a2.
Statement 2 : Forces will be same in electric field.

Q.7

Q.8

Q.9

Q.10

Q.11

Q.12

Q.13

Q.14

Statement 1 : Charges Q1 and Q2 are placed inside and


outside respectively of an uncharged conducting shell,
their separation is r. Then the force on Q1 is zero.
Statement 2 : Lines of force cannot enter conducting shell.
Statement 1 : When a charged comb is brought near a
small piece of paper, it attracts the piece.
Statement 2 : Because the paper becomes charged.
Statement 1 : A point charge is placed in a cavity in a metal
block. If another charge is brought outside the metal, the
charge in the cavity does not feel any electric force
Statement 2 : There is no electric field line in the cavity of
a metal block.
Statement 1 : A small metal ball is suspended in a uniform
electric field with an insulated thread. If high energy X-ray
beam falls on the ball, the ball will be deflected in the electric field.
Statement 2 : X-rays beam falls on the ball, the ball will be
deflected in the magnetic field.
Statement 1 : The tyres of aircrafts are slightly conducting.
Statement 2 : If a conductor is connected to ground, the
extra charge induced on conductor will flow to ground.
Statement 1 : Four point
charges q1, q2, q3 and q4 are
as shown in figure. The flux
over the shown Gaussian surface depends only on charges
q1 and q2.
Statement 2 : Electric field at
all points on Gaussian surface depends only on charges
q1 and q2.
Statement 1 : An insulator does not conduct electricity
usually.
Statement 2 : The number of electrons in an insulator is
very small in comparison to that in a conductor.
Statement 1 : The positive charge particle is placed in
front of a spherical uncharged conductor. The number of
lines of forces terminating on the sphere will be more than
those emerging from it.
Statement 2 : The surface charge density at a point on the
sphere nearest to the point charge will be negative and maximum in magnitude compared to other points on the sphere.
GyaanSankalp
45

Electric charges and fields


EXERCISE - 5
MATCH THE COLUMN TYPE QUESTIONS
Each question contains statements given in two columns which have to be matched. Statements (A, B, C, D) in column I
have to be matched with statements (p, q, r, s) in column II.
Q.1
Match the column :
Column I
Column II
Arrangement
Flux through the entire surface of the body.

(A)

q
(p)
0

(Cube)

(B)

q
(q) 8
0

(hemisphere)

q
(r) 4
0

(C)

(D)

Q.2

Q.3

3q
(s)
0

(sphere)

An electric dipole is placed in an electric field. The column I gives the description of electric field and the angle between the

dipole moment p and the electric field intensity E and the column II gives the effect of the electric field on the dipole. Match
the description in Column I with the statements in column II.
Column I
Column II
(A) Uniform electric field, = 0
(p) force = 0
(B) Electric field due to a point charge, = 0
(q) Torque = 0

(C) Electric field between the two oppositely


(r) p.E 0
charged large plates, = 90
(D) Dipole moment parallel to uniformly
(s) Force 0
charged long wire.
Column II describe graph for charge distribution given in column I match the description.
Column I
Column II
E

(A) Uniformly charged ring

46

GyaanSankalp

(p)

(Electric
field intensities)

Electric charges and fields


V

(B) Infinitely large charge conducting sheet

(Electric
potential)

(q)

(C) Infinite non conducting thin sheet.

(r)
r

(D) Hollow non conducting sphere.

(s)
r

Q.4

Q.5

Q.6

Two points, like charges QA and QB are positioned at points A and B. The
electric field strength to the right of charge QB on the line that passes
through the two charges varies according to a law that is represented
shematically in the figure accompanying the problem without employing a
definite scale. Assume electric field to be positive if its direction coincides
with the positive direction on the x-axis. Distance between the charges is .
Column I
Column II
(A) Charge QA
(p) ve
(B) Charge QB
(q) +ve
2

(C) |QA/QB|

x1
(r)
x

(D) x2

(s) (Q / Q )1/ 3 1
A
B

Match the column


Column I
(A) Like charges repels and unlike attracts.
(B) Numerical value of force between two charges
(C) Methods of charging
(D) Amount of induced charge

Column II
(p) Dr. William Gilbert
(q) Thomos Brown
(r) Coulomb by (torsion balance)
(s) Faraday ice pail exp.

Match the column


Column I
(A) e/m of electron
(B) Charge and mass (indirectly) of electron
and quanta of charge.
(C) Concept of line of force
(D) Highest common factor method

Column II
(p) J.J. Thomson
(q) R.A. millikan (by oil drop exp.)
(r) M. Faraday
(s) Max Plank

GyaanSankalp

47

Electric charges and fields


EXERCISE - 6
PASSAGE BASED QUESTIONS

Q.6

PASSAGE 1 (Q.1-Q.5)
A very large, charged plate floats in deep space. Due to
the charge on the plate, a constant electric field E exists
everywhere above the plate. An object with mass m and
charge q is shot upward from the plate with a velocity v
and an angle . It follows the path shown reaching a height
h and a range R. Assume the effects of gravity to be negligible.

Q.7

Q.8

The net electric field is zero near which point?


(A) A
(B) B
(C) C
(D) D
At which point does the net electric field vector point to
the left?
(A) A
(B) B
(C) C
(D) D
At which point would a small positive charge q feel the
greatest force?
(A) A
(B) B
(C) C
(D) D

E
v
h

R
Q.1

Q.2

Q.3

Q.4

Q.5

Which of the following must be true concerning the object


(a) q must be positive
(b) q must be negative
(c) m must be large
(d) m must be small
Which of the following gives the vertical velocity of the
object in terms of h just before colliding with the plate at
the end of its flight
(a)

2gh

(b)

2Eqh

(c)

2mh
Eq

(d)

2qhE
m

Which of the following is true concerning all objects that


follow the path shown when propelled with a velocity v at
an angle
(a) They must have the same mass
(b) they must have the same charge
(c) they must have the same mass and the same charge
(d) their mass to charge ratios must be the same
Suppose E is 10 N/C, m is 1 kg, q is 1C, v is 100 m/s and
is 30. What is h
(a) 25m
(b) 45m
(c) 80 m
(d) 125
Which of the following is true concerning the flight of the
projectile shown
(a) Increasing the mass m decreases the maximum height h
(b) Increasing the charge q increases the maximum height h
(c) Increasing the mass m decreases the downward acceleration
(d) Increasing the charge q decreases the downward acceleration

PASSAGE 2 (Q.6-Q.8)
Related to the following diagram of two charges, +Q and
4Q.

PASSAGE 3 (Q.9-Q.11)
A thin insulating wire is stretched along the diameter of an
insulated circular loop of radius R. A small bead of mass m
and charge q is threaded on to the wire. Two small identical
charges are tied to the hoop at points opposite to each
other, so that the diameter passing through them is
perpendicular to the thread (see figure). The bead is
released at a point which is a distance x0 from the centre of
the loop. Assume that x0 << R.

Q.9

The resultant force acting on the charged bead is :


(A) F
(C) F

GyaanSankalp

x 20

k 2Qq
x 20

x0
r

(B) F

x0
r

(D) F

k 2Qq
x 20

k 2Qq
x 20

x0
r

x0
r

Q.10 The exact equation of motion of the bead along the thread
is :
(A) m

(C) m

d2x
dt

d2 x
dt

k 2Qq x

2 3/2

k 2Qq x

2 3/2

(B) m

(D) m

d2 x
dt 2

d2 x
dt

kQq x

R2

3/2

k Qq x

R2

3/2

Q.11 The time when will the velocity of the bead vanish for the
first time is :
(A)

48

k 2Qq

2 m R3
k 2Qq

(B)

2 m R3
k Qq

Electric charges and fields


(C)

2 m R3
k 4Qq

(D)

2 2 m R 3
k Qq

PASSAGE 3 (Q.12-Q.14)
In the figure shown mA = mB = 1 kg. Block A is neutral while
qB = 1C. Sizes of A and B are negligible. B is released from
rest at a distance 1.8 m from A. Initially spring is neither
compressed nor elongated.

Q.12 If collision between A and B is perfectly inelastic, what is


velocity of combined mass just after collision.
(A) 6 m/s
(B) 3 m/s
(C) 9 m/s
(D) 12 m/s
Q.13 Equilibrium position of the combined mass is at x = ..... m
(A) 2/9
(B) 1/3
(C) 5/9
(D) 7/9
Q.14 The amplitude of oscillation of the combined mass will be:
(A)

E = 10 N/C

2
m
3

(B)

124
m
3

(D)

106
m
9

K = 18 N/m
smooth

A
x=0

B
x = 1.8m

(C)

x-axis

72
m
9

EXERCISE - 7
SUBJECTIVE QUESTIONS
Q.1

Q.2

Q.3

Q.4

Q.5

Q.6

Two identical conducting of spheres (of negligible radius),


having charges of opposite sign, attract each other with a
force of 0.108 N when separated by 0.5 meter. The spheres
are connected by a conducting wire, which is then removed
(when charge stops flowing) and therefore repel each other
with a force of 0.036 N keeping the distance same. What
were the initial charges on the spheres ?
Two particles A and B having charges of +2.00 106 C
and of 4.00 106 C respectively are held fixed at a
separation of 20.0 cm. Locate the point(s) on the line AB
where the electric field is zero.
Find the flux of the electric field through a spherical surface of radius R
R
due to a charge of
8
18.85 10 C at the cen+
+
tre and another equal
charge at a point 2R away
from the centre.
2R
A clock face has negative charges q, 2q, 3q, ........,12q
fixed at the position of the corresponding numerals on the
dial. The clock hands do not disturb the net field due to
point charges. At what time does the hour hand point in
the same direction is electric field at the centre of the dial.
E0 x
i . Find
The electric field in a region is given by E

the charge contained inside a cubical volume bounded by


the surfaces x = 0, y = 0, y = a, z = 0 and z = a.
Take E0 = 5 103 N/C, = 2 cm. and a = 1 cm.
A solid non conducting sphere of radius R has a nonuniform charge distribution of volume charge density,
r
, where 0 is a constant and r is the distance
R
from the centre of the sphere. Show that
(a) the total charge on the sphere is Q = 0R3 and
(b) the electric field inside the sphere has a magnitude
0

given by, E
Q.7

Q.8

Q.9

KQr 2

R4
An electron beam after being accelerated from rest through
a potential difference of 500V in vacuum is allowed to impinge normally on a fixed surface. If the incident current is
100 A, determine the force exerted on the surface assuming that it brings the electrons to rest.
(e = 1.6 1019C, m = 9.0 1031 kg)
A charge Q is uniformly distributed over a rod of length .
Consider a hypothetical cube of edge with the centre of
the cube at one end of the rod. Find the minimum possible
flux of the electric field through the entire surface of the
cube.
A dipole is placed at origin of coordinate system
as shown in figure, find
the electric field at point
P (0, y).

Q.10 An infinitely long string uniformly charged with a linear


density 1 and a segment of
length uniformly charged
with linear density 2 lie in a
plane at right angles to each
other and separated by a distance r0.
Determine the force with which
these two interact.
Q.11 A very long charged thread (lying in the xy plane) which is
having a linear charge density is having one of its end at
a point P as shown in figure. What is electric field intensity
at point Q.

GyaanSankalp

49

Electric charges and fields


Q.12 Two like charged, infinitely long rod with the same linear
charge density of 3 108 C/cm are 2 cm apart. Find the
electric force per unit length on each rod due to the other
and the work done against that force per unit length to be
done in bringing them closer by 1 cm.

45

45
45

P
r

EXERCISE - 8
PREVIOUS YEAR IIT-JEE QUESTIONS
Q.1

Q.2

A metallic solid sphere is placed in a uniform electric field.


The lines of force follow the path (s) shown in figure as
(1996)
1
A
(A) 1
(B) 2
2
(C) 3
3
C
(D) 4
D
4

Q.4

Q.5

(B) Simple harmonic for all values of satisfying 0 z 0 R


(C) Approximately simple harmonic provided z0 << R
(D) Such that P crosses O and continues to move along
the negative z-axis towards z
A non-conducting solid sphere of radius R is uniformly
charged. The magnitude of the electric field due to the
sphere at a distance r from its centre
(1998)
(A) Increases as r increases for r < R
(B) Decreases as r increases for 0 < r <
(C) Decreases as r increases for R < r <
(D) Is discontinuous at r = R
The dimension of (1/2) 0 E 2 ( 0 : permittivity of free space,
E : electric field) is
(2000)
(A) MLT1
(B) ML2T2
(C) ML1T2
(D) ML2T1
Three positive charges of equal value q are placed at the
vertices of an equilateral triangle. The resulting lines of
force should be sketched as in
(2001)

(A)

50

Q.6

A positively charged thin metal ring of radius R is fixed in


the xy-plane with its centre at the O. A negatively charged
particle P is released from rest at the point (0, 0, z0), where
z0 > 0. Then the motion of P is
(1998)
(A) Periodic for all values of z0 satisfying 0 z 0

Q.3

(C)

GyaanSankalp

(B)

Q.7

Q.8

Q.9

(D)

A small ball of mass 2 103 kg having a charge of 1C is


suspended by a string of length 0.8m. Another identical
ball having the same charge is kept at the point of suspension. Determine the minimum horizontal velocity which
should be imparted to teh lower ball so that it can make
complete revolution.
(2002)
A metallic shell has a point charge q kept inside its cavity.
Which one of the following diagrams correctly represents
the electric lines of forces
(2003)

(A)

(B)

(C)

(D)

A charge +Q is fixed at the origin of the co-ordinate system

while a small electric dipole of dipole-moment p pointing


away from the charge along the x-axis is set free from a
point far away from the origin.
(2003)
(a) Calculate the K.E. of the dipole when it reaches to a
point (d, 0).
(b) Calculate the force on the charge +Q at this moment.
Six charges, three positive and three negative of equal
magnitude are to be placed at the vertices of a regular
hexagon such that the electric field at O is double the elec-

Electric charges and fields


tric field when only one positive charge of same magnitude is placed at R. Which of the following arrangements
of charges is possible for P, Q, R, S, T and U respectively
Q
P
(2004)
(A) +, , +, , , +
(B) +, , +, , +,
U
R
O
(C) +, +, , +, ,
(D) , +, +, , +,
S

Q.10 Consider the charge configuration and spherical Gaussian


surface as shown in the figure. When calculating the flux
of the electric field over the spherical surface the electric
field will be due to
(2004)
(A) q2
q2
(B) only the positive charges
+q1
(C) all the charges
(D) +q1 and q1
-q1
Q.11 Three infinitely long charge sheets are placed as shown in
figure. The electric field at point P is
(2005)
^
k

-2
-

2
(A) k
0

2
(B) k
0

4
(C) k
0

4
(D) k
0

z=a
z=-a
z=-2a

Q.12 Consider a neutral conducting sphere. A positive point


charge is placed outside the sphere. The net charge on the
sphere is then
(2007)
(A) negative and distributed uniformly over the surface of
the sphere
(B) negative and appears only at the point on the sphere
closest to the point charge
(C) negative and distributed non-uniformly over the entire
surface of the sphere
(D) zero
Q.13 A spherical portion has been removed from a solid sphere
having a charge distributed uniformly in its volume as
shown in the figure. The electric field inside the emptied
space is
[2007]
(A) zero everywhere
(B) non-zero and uniform
(C) non-uniform
(D) zero only at its center
Q.14 Positive and negative point charge of equal magnitude are
kept at (0, 0, a/2) and (0, 0, a/2) respectively. The work
done by the electric field when another positive point
charge is moved from (a, 0, 0) to (0, a, 0) is [2007]
(A) positive
(B) negative
(C) zero

(D) depends on the path connecting the initial and final


positions
y
Q.15 Consider a system of three
charges q/3,q/3 and 2q/3
placed at points A, B and
B
C, respectively, as shown
in the figure.
C
x
Take O to be the centre of
O
60
the circle of radius R and
angle CAB = 60
A
[2008]
q

(A) The electric field at point O is

8 0 R 2

directed along

the negative x-axis


(B) The potential energy of the system is zero
(C) The magnitude of the force between the charges at C
q2

and B is

54 0 R 2

q
(D) The potential at point O is 12 R
0

Paragraph for Question Nos. 16 to 18


[2008]
The nuclear charge (Ze) is non-uniformly distributed within
a nucleus of radiusR. Thecharge density (r) [charge per
unit volume] is dependent only on the radial distance r
from the centre of the nucleus as shown in figure The
electric field is only along the radial direction.

Q.16 The electric field at r = R is


(A) independent of a
(B) directly proportional to a
(C) directly proportional to a2
(D) inversely proportional to a
Q.17 For a = 0, the value of d (maximum value of as shown in
the figure) is
(A)

(C)

3Ze
3

(B)

(D)

4R
4Ze

3Ze
R 3
Ze

3R
3R 3
Q.18 The electric field within the nucleus is generally observed
to be linearly dependent on r. This implies.
(A) a = 0
(B) a = R/2
(C) a = R
(D) a = 2R/3
GyaanSankalp

51

Electric charges and fields

HINTS & SOLUTIONS


EXERCISE - 1
This can be done if the end of the rod held in the hand is
coated with an insulating material.
The sphere should be brought in contact with two similar
neutral spheres.
The static charge that may be formed as the car moves is
conducted to the earth through the chain.
Yes, it can.
The surface charge density will increase.
At first the ball will touch the electrically charged rod and
then will be repelled from it.
No, they do not.
This is done for electrostatic shielding.
(a) Yes, they can (b) No, the cannot.
Due to polarization of the insulator rod AB, the point charge
+q1 will be acted upon, in addition to the point charge
q2, by the polarization charges formed at the ends of the
rod.

(1)
(2)
(3)
(4)
(5)
(6)
(7)
(8)
(9)
(10)

The attractive force exerted by the negative charge induced


at the end A will be stronger than the repulsive force exerted
by the positive charge induced at the end B. Thus, the
total force acting on the charge +q1 will increase.

(11)

(12)

(13)

Suppose that there is a field inside the sphere. It is obvious


from considerations of symmetry that in this case
equipotential surfaces must be spherical surfaces
concentric with the charged sphere. Accordingly, the lines
of force coincide with the charged sphere. Accordingly,
the lines of force coincide with the radii, i.e. they must
either begin or end at the centre of teh sphere. This would
have been possible, if there were a positive or a negative
charge at the centre of the sphere. But since there is no
charge inside the sphere, the lines of force cannot begin or
end there. Consequently there is no field inside the sphere.
It can, if we use the phenomenon of electrostatic induction.
Bring a conductor on an insulated support up to the
charged body and connect the conductor to the earth for
a short time. The conductor will retain a charge opposite in
sign to the given one, while the like charge will pass into
the earth.
The charge can be removed from the conductor by
introducing the latter into a metallic space. Teh operation
may be repeated many times with a charge of any
magnitude. Electrostatic machines operate on a similar
principle.
They can, if the charge of one ball is much greater than
that of the other. The forces of attraction caused by the
induced charges may exceed the forces of repulsion.

EX ERCIS E - 2
Q

(2)

10

11

12

13

14

15

16

17

18

19

20

21

22

23

24

25

26

27

28

29

30

31

32

33

34

35

36

37

38

39

40

41

42

43

44

45

46

47

48

49

50

51

52

1 q (Q q)
(D). F 4
r2
0

for F to be maximum,
F

(4)

(D). R 82 82 2.8.8cos120 8N
8N

dF
0
dq

120

q 1

Q 2

60
8N

GyaanSankalp

60

1 1
[Q q q(1)] 0
4 0 r 2

Q 2q = 0

52

Electric charges and fields


(5)

1 (Q / 2) (Q / 2)
(C). 4 . (0.3m 2 ) (100 N / m) (0.1m)
0
Q2

(6)

4 100 (0.1) (0.3) 2

9 109
Q = 20 106 C = 20 C
(A). We have centripetal force equation
2k mv
q

r
r

(7)

(9)
(12)

(15)

so v

2kq
m

where

(17)

a
Now,

2r
m
2r
v
2kq

1
4 0

1 2p
1
1
(B). E 4 3 ; E 3 F 3
r
r
0 r
Hence, the force will become F/8.
(B). U1 will be positive and greatest since all forces among
dipoles are repulsive, U2 is negative as potential energy of
first and second dipole pair cancels out potential energy
of second and third pair, leaving only potential energy of
interaction of first and third, that is negative. In (C), effect
of attraction is greatest.
E
(B). Electric field at
each point on the surR
face of ring due to dipole is

kp

Hence net force on ring is

F QE

(B)
The diagrammatic
representation of the
given problem is
shown in fig.

The electrical field E

q
Bx 2
A

Bx
dx

Ax

0
;
m 0
2

2A
B
From eq. (1) and (2)
x = 0, x =

...(2)

q
A Bx q A B 2A
m
m
B

q
A 2A qA .
m
m
(D). Electric field vector due to a point charge is never
same at two points.
(A) is false
Electric field increases as one goes away from centre of
solid uniformly charged sphere.
This is true till one reaches the surface.
As one moves away from surface (away from centre as
well) the magnitude of electric field increases.
(B) is false
(B). Force on a charge particle in a uniform electric field
F=qE
The acceleration imparted to the particle is
a = qE/m
The distance traveled by the particle in time t is

(19)

(20)

kpQ
d

R3

1 2 1 qE 2
at
t
2
2 m

For the given problem


E

t 2p

t2
e ;
m p me

E Ei
E

The electrical field E

at all points on the Xaxis will not have the


same direction.

...(1)

vdv

in direction
R3
opposite to the dipole moment.

(16)

q
(A Bx)
m

vdv q
q
(A Bx) ; vdv A Bx dx
m
dx
m

(D). The electric field due to the plate is uniform, exerting


no net force on the dipole. The electric field due to the
point charge Q is non-uniform, exerting net force along the
positive y-axis on the dipole, causing it to move in that
direction.

at all points on the Y-axis will be parallel to the X-axis


(i.e. i direction).
The electric potential at the origin due to both the charge
is zero, hence, no work is done in bringing a test charge
from infinity to the origin.
Dipole moment is directed from the q charge to the +q
charge (i.e. x direction).
(C). F=qE = q (A Bx)
ma = q (A Bx)

+q
(-d, 0)

(24)

-q
(d, 0)

(26)

t 2p
t 2e

mp
me

tp
te

mp
me

(C). From (i) A and C both are charged, either positively or


negatively.
From (ii) Both D and E has no charge and from (iii), A is
positively charged.
Therefore from (i), B is negatively charged.

(A). E.A (5i 2j).2i 10
E

GyaanSankalp

53

Electric charges and fields


(B). The given point

P
is at axis of
dipole
2
and at equatorial line

of P dipole so that
field at given point.

(27)

(1,0,2)
(1,0,0)

(32)

P/2
z
P

(B). There exists a point P on the x-axis (other than the


origin), where net electric field is zero. Once the charge Q
reaches point P, attractive forces of the two -ve charge will
dominate and automatically cause the charge Q to cross
the origin.
Now if Q is projected with just enough velocity to reach P,
its K.E. at P is zero, but while being attracted towards origin it acquires KE and hence its net energy at the origin is
positive. (P.E. at origin = zero)

(28)

(D). F1

(31)

q2

Hence,

r2

R r/ K

(C). E.ds
since r << R so we can consider electric field is constant
through out the surface of smaller ring, hence
E

(35)

1 q2
K R2

x
(R 2 x 2 )3 / 2

(A). Let us imagine another identical hemispherical surface


over given one.
Both being symmetric with respect to Q, hence flux will be
same through both the hemisphere (1 = 2).
1 2

1 q2
1 q2
F

,
As F1 = F2
2
4 0 r 2
4 0 K R 2

Q
0

Q
1 2 2
0

EX ERCIS E - 3
Q

10

11

A CD

ABC

ABD

CD

ABC

CD

BC

AD

AC

CD

A B CD

12

13

14

BC

B CD

ABC

(ACD). Q charge must be


induced on the inner surface
of the sheet. Hence +2Q
charge appears on its outer
surface. Hence charge density on the outer surface

(1)

2Q
4R 2

+2Q

+Q
C

(14)

It is evident that at any point outside S is double the field


at a point inside S the field is inversely proportional to the
square of its distance from C as the charge Q is placed at
the centre.
(ABC). Diagonally opposite charges will produce field in
z-axis, but fields due to +ve & ve charges will cancel.

Q
2R 2
EX ERCIS E - 4

(1)

(2)
(3)

(4)
54

10

11

12

13

14

(D). A positive charge brought


in an electric field may increase or decrease the electric field.
At point A electric field increase
at B decrease.
(A). Apply Gauss law.
(B). If +ve charge is displaced along x-axis, then net force
will always act in a direction opposite to that of displacement and the test charge will always come back to its original position.
(A). In conservative electric field total energy remain constant.
GyaanSankalp

(5)

(6)

(A). Electric field inside the conductor is equal to zero,


therefore, potential difference between two position on
the conductor is equal to zero. It means, the potential is
same at every point of the conductor.
(C). qd = e, md = 2mp = 2m
q = 2e, m = 4mp = 4m
F1 = F = eE, F2 = F = 2eE F1
F1
eE

2m 2m

Further,

a1

and a 2

F2
2eE eE

a1
2m 4m 2m

Electric charges and fields


(A). Lines of force from outside charge Q2 cannot penetrate into conducting shell, hence, force on Q1 = 0.
(C). There is redistribution of charge but the net charge
remains zero.
(C). When high energy X-ray beam falls on the ball, the
metal will emit photoelectrons, thus leaving the positive
charge on the ball. As a result of this, ball is deflected in
the direction of electric field.
(B). During take off and landing, the friction between tyres
and the run way may cause electrification of tyres. Due to

(7)
(8)
(10)

(11)

(12)
(13)

(14)

conducting nature of tyre, the charge so collected to a


ground and electrical sparking is avoided
(C). Electric field at any point depends on presence of all
charges.
(C). The number of electrons in insulator is of the same
order as that in a conductor. But the number of free electrons is very small in an insulator. This is basic difference
between a conductor and an insulator.
(D). No. of lines entering the surface = No. of lines leaving
the surface.

EXERCISE - 5
(A) q , (B) p, (C) r, (D) s
(A) p, (B) q (C) r, (D) s
(A) p, (B) r (C) q, (D) s

(4)
(6)
(5)

(A) q, (B) p, (C) p, (D) s


(A) p, q , (B) q, s (C) p, r (D) p, r
(A) p, q, (B) r, (C) r, (D) s

(1)
(2)
(3)

EX ERCIS E - 6
Q

10

11

12

13

14

EXERCISE - 7
(1)
(2)

1.0 106 C , 3 106 C

20
2 1

(7)

Q
(8) 2
0

7.5 109 N

48.3 cm. from A llong BA


kp

(3)

N m2
10
C

(4) 9.39

(5)

2.2 1012 C

2k
,0
(6) 0,
r

(9)

2y

(11)

| E |

(12)

1 2

(10) F 2 ln 1 r

0
0

( i 2j)

2k
, E
r

2k
( j)
r

2k 2
8.1 N / m, 2k 2 , ln 2 0.1129 J / m
r

EXERCISE - 8
(1)
(4)
(7)

(D)
(C)
(C)

(8)

P Q
K.E.
4 0 d 2

(9)
(12)
(15)

(2) (AC)
(5) (C)

(3) (AC)
(6) 5.86 m/s

(b)

2 0 d 3

k (2q / 3)
R2

(A). E

KQ
R2

KZe

R2
So electric field is independent of a.

along positive x-axis

(17)

(B). q

R (R x) 4x

dx Ze

q
6 0 R 2

(Q = Total charge within the nucleus = Ze)

So,

QP

(D)
(10) (C)
(11) (B)
(ABCD)
(13) (B)
(14) (C)
(C). Net electric field to both charges q/3, will get cancelled.
Electric field due to (2q/3) will be directed in ve axis.
E

(16)

(18)

3Ze

R 3
(C). If within a sphere is constant E r

q 2q
q 2q
K K
2
P.E. of system = K (q / 3) 3 3 3 3
2R
2R sin 60 2R cos 60
GyaanSankalp

55

Vous aimerez peut-être aussi